You are on page 1of 60

PHYSICS

S.No. CONTENTS Page

1. Heat 01

2. Thermodynamics 08

3. Wave Theory 15

4. Ray Optics 21

5. Wave Optics 29

6. Photo Electric Effect & Matter Wave 34

7. Nuclear Physics & Radioactivity 42

8. Semiconductor, POC & Practical Physics 49

E
Heat
ALLEN
HEAT EXERCISE-I
TEMPERATURE & THERMAL EXPANSION CALORIMETRY
1. At what temperature does the temperature in 6. A bo dy of mass 5 kg falls from a h eight of
Celsius and Fahrenheit equalise 30 metre. If its all mechanical energy is changed
(1) –40° (2) 40° into heat, then heat produced will be:-
(3) 36.6° (4) 38° (1) 350 cal (2) 150 cal
2. A difference of temperature of 25° C is equivalent (3) 60 cal (4) 6 cal
to a difference of : 7. The amount of heat required in converting 1 gm
(1) 45° F (2) 72° F ice at -100 C into steam at 1000 C will be :–
(3) 32° F (4) 25° F (1) 3028 J (2) 6056 J
3. The graph AB shown in figure is a plot of (3) 721 J (4) 616 J
temperature of a body in degree Celsius and degree 8. A block of mass 2.5 kg is heated to temperature of
Fahrenheit. Then 500°C and placed on a large ice block. What is the
maximum amount of ice that can melt (approx.).
Specific heat for the body = 0.1 cal/g°C.
100°C B
(1) 1 kg (2) 1.5 kg (3) 2 kg (4) 2.5 kg
9. 1 kg of ice at – 10°C is mixed with 4.4 kg of water
Centigrade

at 30°C. The final temperature of mixture is :


(specific heat of ice = 2100 J/kg-k)
32°F 212°F Fahrenheit
A
(1) 2.3°C (2) 4.4°C (3) 5.3°C (4) 8.7°C
CONDUCTION AND CONVECTION
(1) slope of line AB is 9/5
& RADIATION
(2) slope of line AB is 5/9
(3) slope of line AB is 1/9 10. A composite rod made of three rods of equal length
(4) slope of line AB is 3/9 and cross-section as shown in the fig. The thermal
4. If two rods of length L and 2L having coefficients of conductivities of the materials of the rods are K/2,
linear expansion a and 2a respectively are 5K and K respectively. The end A and end B are at
connected so that total length becomes 3L, the constant temperatures. All heat entering the face
average coefficient of linear expansion of the A goes out of the end B and there being no loss of
composition rod equals: heat from the sides of the bar. The effective thermal
conductivity of the bar is
3 5
(1) a (2) a A B
2 2
K/2 5K K
5
(3) a (4) none of these (1) 15K/16 (2) 6K/13
3
(3) 5K/16 (4) 2K/13.
5. The table gives the initial length l 0, change in
node06\B0AI-BO\Kota\JEE(MAIN)\Booster Course Sheet\Phy\Eng\Heat, Wave & Modern Physics\01-Heat.p65

11. Th e f igure sh ow s t he face and in te rfa ce


temperature DT and change in length Dl of four temperature of a composite slab containing of four
rods. Which rod has greatest coefficient of linear layers of two materials having identical thickness.
expansion Under steady state condition, find the value of
temperature q.
Rod l0(m) DT(°C) Dl(m)

A1 1 100 1

A2 1 100 2

A3 1.5 50 3
A4 2.5 20 4
(1) 5°C (2) 10°C
(1) A1 (2) A2 (3) A3 (4) A4 (3) –15°C (4) 15°C

E 1
JEE ( Main) - Physics
ALLEN
12. Two identical square rods of metal are welded end 20. At a given temperature, the pressure of an ideal
to end as shown in figure (1), 20 calories of heat gas of density r is proportional to -
flows through it in 4 minutes. If the rods are welded
as shown in figure (2), the same amount of heat will 1 1
(1) (2) (3) r2 (4) r
flow through the rods in - r2 r
21. On increasing the temperature of a gas filled in a
closed container by 10C its pressure increases by
0°C 0°C 100°C
100°C 0.4%, then initial temperature of the gas is-
l l
l (1) 250C (2) 2500C
(a) (b) (3) 250 K (4) 25000C
22. The equation of state for 5g of oxygen at a pressure
(1) 1 minute (2) 2 minutes P and temperature T occupying a volume V, will
(3) 4 minutes (4) 16 minutes be :– (where R is the gas constant)
13. The dimensional formula for thermal resistance is (1) PV = 5 RT (2) PV = (5/2) RT
(1) M–1L–2T3q (2) M–1L–2T–3q (3) PV = (5/16) RT (4) PV = (5/32)RT
(3) ML T q
2 –2
(4) ML2T2q–1 23. At constant pressure hydrogen is having temperature
14. A spherical body of area A, and emissivity e = 0.6 of 327° C. Till what temperature it is to be cooled so
is kept inside a black body. What is the rate at that the rms velocity of its molecules becomes half of
which energy is radiated per second at temperature T the earlier value :-
(1) 0.6 s AT4 (2) 0.4 s AT4 (1) –123°C (2) 123°C (3) –100°C (4) 0°C
(3) 0.8 s AT4 (4) 1.0 s AT4 24. The rms velocity of H2 is 2 x 103 m /sec. What will
15. If temperature of ideal black body increased by be the rms velocity of O2 molecules at the same
10%, then percentage increase in quantity of temperature :-
radiation emitted from it's surface will be :- (1) 103 m/sec. (2) 500 m/sec.
(1) 10% (2) 40% (3) 46% (4) 100% (3) 0.5 × 104 m/sec. (4) 3 × 103 m / sec
16. There is a black spot on a body. If the body is heated 25. The pressure exerted by a gas in P0. If the mass of
and carried in a dark room then it glows more. This molecules becomes half and their velocities become
can be explained on the basis of – double, then pressure will become
(1) Newton's law of cooling
P0
(2) Wein's law (1) (2) P0 (3) 2P0 (4) 4P0
2
(3) Kirchhoff's law
(4) Stefan's law 26. According to Maxwell's law of distribution of
17. Solar constant for earth is 2 cal/min cm2, if distance velocites of molecules, the most probable velocity
of mercury from sun is 0.4 times than distance of is :-
earth from sun then solar constant for mercury will (1) greater than the mean velocity
be? (2) equal to the mean velocity
(1) 12.5 cal/min cm2 (2) 25 cal/min cm2 (3) equal to the root mean square velocity
(3) 0.32 cal/min cm2 (4) 2 cal/min cm2 (4) less than the root mean square velocity
node06\B0AI-BO\Kota\JEE(MAIN)\Booster Course Sheet\Phy\Eng\Heat, Wave & Modern Physics\01-Heat.p65

27. Two monoatomic ideal gas at temperature T1 and


KINETIC THEORY OF GASES
T2 are mixed. There is no loss of energy. If the
18. The latent heat for vapourisation for 1 gm water is mass of molecules of the two gases are m1 and m2
536 cal. Its value in Joule/kg will be :- and number of their molecules are n 1 and n 2
(1) 2.25 × 106 (2) 2.25 × 103 respectively, then temperature of the mixture will
(3) 2.25 (4) None of these be :
19. During an experiment an ideal gas obeys an addition
T1 + T2 T1 T2
equation of state P2V = constant. The initial (1) n + n (2) n + n
temperature and pressure of gas are T and V 1 2 1 2

respectively. When it expands to volume 2V, then


n2 T1 + n1 T2 n1 T1 + n2 T2
its temperature will be : (3) (4)
n1 + n2 n1 + n2
(1) T (2) 2 T (3) 2 T (4) 2 2 T

2 E
Heat
ALLEN
28. Relation between the ratio of specific heats (g) of 36. Two balloons are filled, one with pure He gas and
gas and degree of freedom 'f'' will be the other by air, respectively. If the pressure and
temperature of these balloons are same then the
1 1 1
(1) g = f + 2 (2) = + number of molecules per unit volume is :-
g f 2
(1) more in the He filled balloon
(3) f = 2 / (g-1) (4) f = 2( g-1) (2) same in both balloons
29. Relation between pressure (P) and energy density (3) more in air filled balloon
(E) of an ideal gas is -
(4) in the ratio of 1 : 4
(1) P = 2/3 E (2) P = 3/2 E
37. Steam at 100°C is passed into 20 g of water at
(3) P = 3/5 E (4) P = E
30. On mixing 1 gm mole of a monoatomic with 1 gm 10°C. When water acquires a temperature of 80°C,
mole of a diatomic gas the specific heat of mixture the mass of water present will be :
at constant volume will be :– [Take specific heat of water = 1 cal g–1 °C–1 and
(1) R (2) 3/2 R (3) 2R (4) 5/2R latent heat of steam = 540 cal g–1]
31. If 2 gm moles of a diatomic gas and 1 gm mole of (1) 24 g (2) 31.5 g
a mono-atomic gas are mixed then the value of (3) 42.5 g (4) 22.5 g
g (Cp/Cv) for mixture will be :–
CP
13 19 7 5
38. The ratio of the specific heats = g in terms of
(1) (2) (3) (4) CV
19 13 5 3
degrees of freedom (n) is given by :
32. For a diatomic gas, change in internal energy for
unit change in temperature at constant pressure æ nö æ 2ö
and volume is U1 and U2 respectively then U1 : U2 (1) ç 1 + ÷ (2) ç 1 + ÷
è 3ø è nø
is :
(1) 5 : 3 (2) 7 : 5 (3) 1 : 1 (4) 5 : 7 æ nö æ 1ö
33. The specific heat of a gas : (3) ç 1 + ÷ (4) ç 1 + ÷
è 2ø è nø
(1) Has only two value Cp and Cv
(2) Has a unique value at a given temperature 39. A slab of stone of area 0.36 m2 and thickness
(3) Can have any value between 0 and ¥ 0.1 m is exposed on the lower surface to steam at
100°C. A block of ice at 0°C rests on the upper
(4) Depends upon the mass of the gas
surface of the slab. In one hour 4.8 kg of ice is
34. 22 gm. of CO2 at 270 C is mixed with 16 gm. of
O2 at 370 C. The temperature of the mixture is :– melted. The thermal conductivity of slab is :
(At room temperature, degrees of freedom of (Given latent heat of fusion of ice 3.36 ×105 J kg–1)
CO2=7 and degrees of freedom of O2 = 5) (1) 2.05 J/m/s/°C (2) 1.02 J/m/s/°C
(1) 31.160 C (2) 270 C
(3) 370 C (4) 300 C (3) 1.24 J/m/s/°C (4) 1.29 J/m/s/°C
35. Oxygen and hydrogen gases are at temperature 40. What is the slope for an isothermal process in PV
T. Then K.E of molecules of oxygen gas is equal to indicator diagram :-
node06\B0AI-BO\Kota\JEE(MAIN)\Booster Course Sheet\Phy\Eng\Heat, Wave & Modern Physics\01-Heat.p65

how many times of K.E. of molecules of


hydrogen gas :- P P
(1) (2) – (3) Zero (4) ¥
(1) 16 times (2) 8 times V V
(3) Equal (4) 1/16 times
ANSWER KEY Exercise-I
Que. 1 2 3 4 5 6 7 8 9 10
Ans. 1 1 2 3 4 1 1 2 4 1
Que. 11 12 13 14 15 16 17 18 19 20
Ans. 1 1 1 1 3 3 1 1 2 4
Que. 21 22 23 24 25 26 27 28 29 30
Ans. 3 4 1 2 3 4 4 3 1 3
Que. 31 32 33 34 35 36 37 38 39 40
Ans. 2 3 3 1 3 2 4 2 4 3

E 3
JEE ( Main) - Physics
ALLEN
PREVIOUS YEARS' QUESTIONS EXERCISE-II
7. Two thermally insulated vessels 1 and 2 are filled
1. Heat given to a body which raises its temperature
with air at temperatures (T1, T2), volume (V1, V2)
by 1°C is- [AIEEE - 2002]
and pressure (P1, P2) respectively. If the valve
(1) water equivalent joining the two vessels is opened, the temperature
(2) thermal capacity inside the vessel at equilibrium will be
[AIEEE - 2004]
(3) specific heat
(1) T1 + T2
(4) temperature gradient
( T1 + T2 )
2. Cooking gas containers are kept in a lorry moving (2)
2
with uniform speed. The temperature of the gas
molecules inside will- [AIEEE - 2002]
T1 T2 (P1 V1 + P2 V2 )
(3)
(1) increase P1 V1 T2 + P2 V2 T1

(2) decrease
T1 T2 (P1 V1 + P2 V2 )
(3) remains same (4)
P1 V1 T1 + P2 V2 T2
(4) decrease for some, while increase for others
8. If the temperature of the sun were to increase from
3. At what temperature is the rms velocity of a T to 2T and its radius from R to 2R, then the ratio
hydrogen molecule equal to that of an oxygen of the radiant energy received on earth to what
molecules at 47° C ? [ AIE EE -2002] it was previously, will be- [AIEEE - 2004]

(1) 80 K (2) – 73 K (1) 4 (2) 16 (3) 32 (4) 64


(3) 3 K (4) 20 K 9. The temperature of the two outer surfaces of a
4. 1 mole of a gas with g = 7/5 is mixed with 1 mole composite slab, consisting of two materials having
of a gas with g = 5/3, then the value of g for the coefficients of thermal conductivity K and 2K and
resulting mixture is- [AIE EE-2002] thickness x and 4x, respectively are T 2 and
T1(T2 > T1). The rate of heat transfer through the
(1) 7/5 (2) 2/5
æ A(T2 - T1 )K ö
(3) 24/16 (4) 12/7 slab, in a steady state is ç ÷ f, with f
è x ø
5. Two spheres of the same material have radii
equals to- [AIEEE - 2004]
1 m and 4 m and temperatures 4000 K and
2000 K respectively. The ratio of the energy (1) 1 x 4x

radiated per second by the first sphere to that by


(2) 1/2 T2 K 2K T1
node06\B0AI-BO\Kota\JEE(MAIN)\Booster Course Sheet\Phy\Eng\Heat, Wave & Modern Physics\01-Heat.p65

the second is- [AIEEE - 2002]

(1) 1 : 1 (2) 16 : 1 (3) 2/3

(3) 4 : 1 (4) 1 : 9 (4) 1/3

6. One mole of ideal monoatomic gas (g = 5/3) is 10. A gaseous mixture consists of 16 g of helium and
mixed with one mole of diatomic gas (g = 7/5).
Cp
What is g for the mixture ? g denotes the ratio of 16 g of oxygen. The ratio of the mixture
Cv
specific heat at constant pressure, to that at constant
volume- [AIEEE-2004] is- [AIEEE-2005]

(1) 3/2 (2) 23/15 (1) 1.59 (2) 1.62


(3) 35/23 (4) 4/3 (3) 1.4 (4) 1.54

4 E
Heat
ALLEN
11. The figure shows a system of two concentric spheres 14. If CP and CV denote the specific heats of nitrogen
of radii r1 and r2 and kept at temperatures T1 per unit mass at constant pressure and constant
and T2, respectively. The radial rate of flow of volume respectively, then- [AIEEE - 2007]
heat in a substance between the two concentric
(1) CP – CV = R/28 (2) CP – CV = R/14
spheres, is proportional to- [AIEEE - 2005]
(3) CP – CV = R (4) CP – CV = 28 R

15. One end of a thermally insulated rod is kept at


a temperature T1 and the other at T2. The rod
r1
is composed of two sections of lengths l1 and l2
• T1 and thermal conductivities K1 and K2 respectively.
r2 The temperature at the interface of the two sections
T2
is- [AIEEE - 2007]

(r2 - r1 ) æ r2 ö T1 l1 l2
(1) (2) ln ç ÷ T2
(r1r2 ) è r1 ø

r1r2
(3) (4) (r2 – r1)
(r2 - r1) K1 K2

12. Two rigid boxes containing different ideal gases


are placed on a table. Box A contains one mole (1) (K2l2T1 + K1l1T2)/(K1l1 + K2l2)
of nitrogen at temperature. T 0, while box B (2) (K2l1T1 + K1l2T2)/(K2l1 + K1l2)
contains one mole of helium at temperature
(3) (K1l2T1 + K2l1T2)/(K1l2 + K2l1)
(7/3) T0. The boxes are then put into thermal
contact with each other, and heat flows between (4) (K1l1T1 + K2l2T2)/(K1l1 + K2l2)
them until the gases reach a common final 16. An insulated container of gas has two chambers
temperature (Ignore the heat capacity of boxes). separated by an insulating partition. One of the
Then, the final temperature of gases, T f, in terms chambers has volume. V1 and contains ideal gas
of T0 is - [AIEEE - 2006]
at pressure P1 and temperature T1. The chamber
has volume V2 and contains ideal gas at pressure
3 7
(1) Tf = T (2) Tf = T P2 and temperature T2. If the partition is removed
7 0 3 0
without doing any work on the gas, the final
equilibrium temperature of the gas in the container
3 5 will be [AIEEE - 2008]
(3) Tf = T (4) Tf = T
2 0 2 0
T1 T2 (P1 V1 + P2 V2 ) P1 V1 T1 + P2 V2 T2
13. Assuming the sun to be a spherical body of radius (1) (2)
node06\B0AI-BO\Kota\JEE(MAIN)\Booster Course Sheet\Phy\Eng\Heat, Wave & Modern Physics\01-Heat.p65

P1 V1 T2 + P2 V2 T1 P1 V1 + P2 V2
R at a temperature of T K, evaluate the total radiant
power, incident on earth, at a distance r from the
P1 V1 T2 + P2 V2 T1 T1 T2 (P1 V1 + P2 V2 )
sun- (r0 is the radius of earth) [AIEEE - 2006] (3) (4) P V T + P V T
P1 V1 + P2 V2 1 1 1 2 2 2

(1) 4pr02 R2 sT4 /r2


17. One kg of a diatomic gas is at a pressure of
8 × 104 N/m2. The density of the gas is 4 kg/m3.
(2) pr02 R2 sT4/r2
What is the energy of the gas due to its thermal
motion ? [AIEEE - 2009]
(3) r02 R2 sT4 / 4pR2
(1) 6 × 104 J (2) 7 × 104 J
(4) R2 sT4/r2 (3) 3 × 104 J (4) 5 × 104 J

E 5
JEE ( Main) - Physics
ALLEN
18. A long metallic bar is carrying heat from one of its 21. A wooden wheel of radius R is made of two
ends to the other end under steady-state. The semicircular parts (see figure). The two parts are
variation of temperature q along the length x of the held together by a ring made of a metal strip of cross
bar from its hot end is best described by which of sectional area S and Length L. L is slightly less than
the following figures ? [AIEEE - 2009] 2pR. To fit the ring on the wheel, it is heated so that
its temperature rises by DT and it just steps over the
wheel. As it cools down to surrounding temperature,
q q
it presses the semicircular parts together. If the
coefficient of linear expansion of the metal is a, and
(1) (2)
its Young's modulus is Y, the force that one part of
x
x the wheel applies on the other part is :

[AIEEE - 2012]

q
(1) 2SYaDT
q
(2) 2 p SYaDT R

(3) (4) (3) SYaDT


x x
(4) p SYaDT

22. A liquid in a beaker has temperature q(t) at time t


19. Three perfect gases at absolute temperatures T1, and q 0 is temperature of surroundings, then
T2 and T3 are mixed. The masses of molecules are according to Newton's law of cooling the correct
m1, m2, and m3 and the number of molecules are graph between loge (q – q0) and t is :- [AIEEE - 2012]
n1, n2 and n3 respectively. Assuming no loss of
energy, then final temperature of the mixture is :-

loge(q –q 0)
loge(q –q 0)

[AIEEE - 2011]

n1 T12 + n 2 T22 + n 3 T32 (1) (2)


(1)
n1 T1 + n 2 T2 + n 3 T3 0
t
t

n12 T12 + n 22 T22 + n 32 T32


(2)
n1 T1 + n 2 T2 + n 3 T3
loge(q –q 0)

loge(q –q 0)

T1 + T2 + T3
(3) (3) (4)
3 t
node06\B0AI-BO\Kota\JEE(MAIN)\Booster Course Sheet\Phy\Eng\Heat, Wave & Modern Physics\01-Heat.p65

n1T1 + n 2 T2 + n 3 T3
(4) 23. The pressure that has to be applied to the ends of
n1 + n 2 + n 3
a steel wire of length 10 cm to keep its length
constant when its temperature is raised by 100°C is:
20. An aluminium sphere of 20 cm diameter is heated
from 0°C to 100°C. Its volume changes by (given (For steel Young's modulus is 2 × 1011 N m–2
that coefficient of linear expansion for aluminium and coefficient of thermal expansion is 1.1 ×10–5 K–1 )
aAl = 23 × 10–6/°C :- [AIEEE - 2011]
[JEE(Main)-2014]
(1) 28.9 cc (2) 2.89 cc
(1) 2.2 × 107 Pa (2) 2.2 × 106 Pa
(3) 9.28 cc (4) 49.8 cc
(3) 2.2 × 108 Pa (4) 2.2 × 109 Pa

6 E
Heat
ALLEN
24. Three rods of Copper, Brass and Steel are welded 27. Cp and Cv are specific heats at constant pressure
together to form a Y-shaped structure. Area of and constant volume respectively. It is observed that
cross-section of each rod = 4 cm2. End of copper
Cp – Cv = a for hydrogen gas
rod is maintained at 100°C where as ends of brass
and steel are kept at 0°C. Lengths of the copper, Cp – Cv = b for nitrogen gas
brass and steel rods are 46, 13 and 12 cms The correct relation between a and b is :
respectively. The rods are thermally insulated from
[JEE(Main)-2017]
surroundings except at ends. Thermal conductivities
of copper, brass and steel are 0.92, 0.26 and (1) a = 14 b (2) a = 28 b
0.12 CGS units respectively. Rate of heat flow
through copper rod is : [JEE(Main)-2014] 1
(3) a = b (4) a = b
14
(1) 4.8 cal/s (2) 6.0 cal/s

(3) 1.2 cal/s (4) 2.4 cal/s 28. A copper ball of mass 100 gm is at a temperature
T. It is dropped in a copper calorimeter of mass
25. A pendulum clo ck loses 12s a day if t he
100 gm, filled with 170 gm of water at room
temperature is 40°C and gains 4s a day if the
temperature. Subsequently, the temperature of the
temperature is 20°C. The temperature at which the
system is found to be 75°C. T is given by :
clock will show correct time, and the co-effecient of
(Given : room temperature = 30° C, specific heat
linear expansion (a) of the metal of the pendulum
of copper = 0.1 cal/gm°C [JEE(Main)-2017]
shaft are respectively :- [JEE(Main)-2016]
(1) 1250°C (2) 825°C
(1) 55°C ; a = 1.85 × 10–2 / °C
(3) 800°C (4) 885° C
(2) 25°C ; a = 1.85 × 10–5 / °C
29. An external pressure P is applied on a cube at 0°C
(3) 60°C ; a = 1.85 × 10–4 / °C so that it is equally compressed from all sides.
K is the bulk modulus of the material of the cube
(4) 30°C ; a = 1.85 × 10–3 / °C
and a is its coefficient of linear expansion. Suppose
26. The temperature of an open room of volume we want to bring the cube to its original size by
30 m3 increases from 17°C to 27°C due to sunshine. heating. The temperature should be raised by :
The atmospheric pressure in the room remains
[JEE(Main)-2017]
1 × 105 Pa. If ni and nf are the number of molecules
in the room before and after heating, then nf – ni 3a
(1) (2) 3PKa
will be :- [JEE(Main)-2017] PK
(1) 2.5 × 1025 (2) –2.5 × 1025 P P
(3) (4)
(3) –1.61 × 1023 (4) 1.38 × 1023 3aK aK
node06\B0AI-BO\Kota\JEE(MAIN)\Booster Course Sheet\Phy\Eng\Heat, Wave & Modern Physics\01-Heat.p65

PREVIOUS YEARS QUESTIONS ANSWER KEY Exercise-II


Que. 1 2 3 4 5 6 7 8 9 10
Ans. 2 3 4 3 1 1 3 4 4 2
Que. 11 12 13 14 15 16 17 18 19 20
Ans. 3 3 2 1 3 1 4 4 4 1
Que. 21 22 23 24 25 26 27 28 29
Ans. 1 2 3 1 2 2 1 4 3

E 7
JEE ( Main) - Physics
ALLEN
THERMODYNAMICS EXERCISE-I

1. The first law of thermodynamics is based on :– 6. In a cyclic process shown on the P – V diagram, the
magnitude of the work done is :
(1) Law of conservation of energy
P
(2) Law of conservation of mechanical energy
P2
(3) Law of conservation of gravitational P.E.
P1
(4) None of the above
V
O
2. In a process, 500 calories of heat is given to a system V1 V2

and at the same time 100 joules of work is done on


2 2
the system. The increase in the internal energy of æ P2 - P1 ö æ V2 - V1 ö
(1) p ç ÷ (2) p ç ÷
the system is :- è 2 ø è 2 ø
(1) 40 calories (2) 1993 joules
p
(3) (P – P1) (V2 – V1) (4) p (P2V2 – P1V1)
(3) 2193 joules (4) 82 calories 4 2

3. In a thermodynamic process pressure of a fixed 7. A system is taken along the paths A and B as shown.
mass of a gas is changed in such a manner that the If the amounts of heat given in these processes are
gas releases 20 joules of heat and 8 joules of work DQA and DQB and change in internal energy are
was done on the gas. If the initial internal energy of DUA and DUB respectively then :-
the gas was 30 joules, then the final internal energy
will be:-

(1) 2 J (2) 42 J (3) 18 J (4) 58 J A


P i f
4. When a system is taken from state ‘a’ to state
‘b’ along the path ‘acb’, it is found that a quantity B
of heat Q = 200 J is absorbed by the system and V
a work W = 80J is done by it. Along the path
‘adb’, Q = 144J. The work done along the path
‘adb’ is (1) DQA = DQB; DUA < DUB

(2) DQA ³ DQB; DUA = DUB


p
c b (3) DQA < DQB; DUA > DUB

(4) DQA > DQB; DUA = DUB


node06\B0AI-BO\Kota\JEE(MAIN)\Booster Course Sheet\Phy\Eng\Heat, Wave & Modern Physics\02-Thermodynamics.p65

8. Which of the following graphs correctly represents


a d
V the variation of b = –(dV/dP)/V with P for an ideal
gas at constant temperature?
(1) 6J (2) 12 J

(3) 18 J (4) 24 J

5. 1 kg of a gas does 20 kJ of work and receives (1) (2)


16 kJ of heat when it is expanded between two
states. A second kind of expansion can be found
between the same initial and final state which
requires a heat input of 9 kJ. The work done by
the gas in the second expansion is : (3) (4)
(1) 32 kJ (2) 5 kJ (3) –4 kJ (4) 13 kJ
8 E
Thermodynamics
ALLEN
9. P-V plots for two gases during adiabatic processes 15. Graph of isometric process is :-
are shown in the figure. Plots 1 and 2 should
correspond respectively to

(1) P (2) P

T T

(1) He and O2 (2) O2 and He (3) P (4) P


(3) He and Ar (4) O2 and N2
T
10. For an adiabatic expansion of a perfect gas, the T
value of DP/P is equal to:- 16. According to the second law of thermodynamics :

(1) – g DV/V (2) –DV/V (1) heat energy cannot be completely converted
to work
(3) –g DV/V (4) –g2 DV/V
11. A gas for which g = 5/3 is heated at constant (2) work cannot be completely converted to heat
pressure. The percentage of total heat given that energy
will be used for external work is : (3) for all cyclic processes we have dQ/T < 0

(1) 40% (2) 30% (3) 60% (4) 20% (4) the reason all heat engine efficiencies are less
than 100% is friction, which is unavoidable
12. One mole of an ideal gas at temperature T1 expends
17. A reversible refrigerator operates between a low
P temperature reservoir at TC and a high temperature
according to the law 2 = a (constant). The work
V reservoir at TH. Its coefficient of performance is
done by the gas till temperature of gas becomes T2 given by :
is : (1) (TH – TC)/TC (2) TC/(TH – TC)
1 1 (3) (TH – TC)/TH (4) TH/(TH – TC)
(1) R(T2 – T1) (2) R(T2 – T1)
2 3
18. The efficiency of carnot engine is 50% and
temperature of sink is 500K. If temperature of
1 1
(3) R(T2 – T1) (4) R(T2 – T1) source is kept constant and its efficiency raised to
4 5
60%, then the required temperature of the sink
13. A quantity of air (g = 1.4) at 27°C is compressed
node06\B0AI-BO\Kota\JEE(MAIN)\Booster Course Sheet\Phy\Eng\Heat, Wave & Modern Physics\02-Thermodynamics.p65

will be :-
suddenly, the temperature of the air system will :
(1) 100 K (2) 600 K (3) 400 K (4) 500 K
(1) Fall (2) Rise
19. A refrigerator works between temperature –10°C
(3) Remain unchanged (4) First rise and then fall
and 27°C, the coefficient of performance is :
æ 4ö (1) 7.1 (2) 1 (3) 8.1 (4) 15.47
14. The volume of a poly-atomic gas ç g = ÷
è 3ø
20. An ideal gas expands isothermally from a volume
V1 to V2 and then compressed to original volume
1
compressed adiabatically to of the original V1 adiabatically. Initial pressure is P1 and final
8 th
pressure is P3. The total work done is W. Then
volume. If the original pressure of the gas is P 0 the
new pressure will be : (1) P3 > P1, W > 0 (2) P3 < P1, W < 0

(1) 8 P0 (2) 16 P0 (3) 6 P0 (4) 2 P0 (3) P3 > P1, W < 0 (4) P3 = P1, W = 0

E 9
JEE ( Main) - Physics
ALLEN
21. An ideal gas is taken through the cycle A ® B ® C ® A, 23. Th e above P-V diag ram represen ts t he
as shown in the figure. If the net heat supplied to thermodynamic cycle of an engine, operating with
the gas in the cycle is 5J, the work done by the gas an ideal monoatomic gas. The amount of heat,
in the process C ® A is extracted from the source in a single cycle is :

2p0 B C

p0 D
(1) –5J (2) –10 J
A

(3) –15 J (4) –20 J V


v0 2v0
22. Three processes form a thermodynamic cycle as
shown on P-V diagram for an ideal gas. Process
13
1 ® 2 takes place at constant temperature (300K). (1) p0v0 (2) æç ö÷ p0 v 0
è 2 ø
Process 2 ® 3 takes place at constant volume.
During this process 40J of heat leaves the system. æ 11 ö
(3) ç ÷ p0 v 0 (4) 4p0v0
Process 3 ® 1 is adiabatic and temperature T3 is è 2 ø
275K. Work done by the gas during the process 24. In the above question efficiency of cycle ABCDA is
3 ®1 is nearly :

(1) 12.5% (2) 15.4%

(3) 9.1% (4) 10.5%

(1) -40J (2) -20J

(3) +40J (4) +20J

node06\B0AI-BO\Kota\JEE(MAIN)\Booster Course Sheet\Phy\Eng\Heat, Wave & Modern Physics\02-Thermodynamics.p65

ANSWER KEY Exercise-I


Que. 1 2 3 4 5 6 7 8 9 10
Ans. 1 3 3 4 4 3 4 1 2 3
Que. 11 12 13 14 15 16 17 18 19 20
Ans. 1 2 2 2 3 1 2 3 1 3
Que. 21 22 23 24
Ans. 1 1 2 2

10 E
Thermodynamics
ALLEN
PREVIOUS YEARS' QUESTIONS EXERCISE-II
1. During an adiabatic process, the pressure of a gas 7. Which of the following statements is correct for
is found to be proportional to the cube of its any thermodynamic system ? [AIEEE - 2004]
absolute temperature. The ratio CP/CV for the gas (1) The internal energy changes in all processes
is- [AIEEE - 2002]
(2) Internal energy and entropy are state functions
(1) 4/3 (2) 2 (3) The change in entropy can never be zero
(3) 5/3 (4) 3/2 (4) The work done in an adiabatic process is always
2. If mass-energy equivalence is taken into account, zero
when water is cooled to form ice, the mass of water 8. Which of the following is incorrect regarding the
should- [AIEEE - 2002] first law of thermodynamics ? [AIEEE - 2005]

(1) increase (1) It is applicable to any cyclic process


(2) It is a restatement of the principle of conservation
(2) remain unchanged
of energy
(3) decrease
(3) It introduces the concept of the internal energy
(4) first increase then decrease (4) It introduced the concept of the entropy
3. Even carnot engine cannot give 100% efficiency 9. The temperature-entropy diagram of a reversible
because we cannot- [AIEEE - 2002] engine cycle is given in the figure. Its efficiency
(1) prevent radiation is- [AIEEE - 2005]

(2) find ideal sources


T
(3) reach absolute zero temperature
2T0 •
(4) eliminate friction
4. "Heat cannot be itself flow from a body at lower T0 •
temperature to a body at higher temperature" is • • S
S0 2S0
a statement or consequence of- [AIEEE - 2003]
(1) second law of thermodynamics (1) 1/2 (2) 1/4 (3) 1/3 (4) 2/3
(2) conservation of momentum 10. A system goes from A to B via two processes I
(3) conservation of mass and II as shown in figure. If DU1 and DU2 are
the changes in internal energies in the processes
(4) first law of thermodynamics
I and II respectively then- [AIEEE - 2005]
5. Which of the following parameters does not
characte rise th e the rmodyn amic st at e of P
node06\B0AI-BO\Kota\JEE(MAIN)\Booster Course Sheet\Phy\Eng\Heat, Wave & Modern Physics\02-Thermodynamics.p65

matter ? [AIEEE - 2003]

(1) Temperature (2) Pressure II


(3) Work (4) Volume A• •B
I
6. A carnot engine takes 3 × 106 cal of heat from
a reservoir at 627 °C and gives it to a sink at
V
27 °C. The work done by the engine is-
[AIEEE - 2003]
(1) DU1 = DU2
(1) 4.2 × 106 J
(2) relation between DU1 and DU2 cannot be
(2) 8.4 × 106 J determined
(3) 16.8 × 106 J (3) DU2 > DU1
(4) zero (4) DU2 < DU1

E 11
JEE ( Main) - Physics
ALLEN
11. The work of 146 kJ is performed in order to 15. The work done on the gas in taking it from D to A
compress one kilo mole of a gas adiabatically and is :- [AIEEE - 2009]
in this process the temperature of the gas increases
(1) –690 R (2) +690 R
by 7°C. The gas is- (R = 8.3 J mol–1 K–1)
(3) –414 R (4) +414 R
[AIEEE - 2006]

(1) diatomic 16. The net work done by the gas in the cycle ABCDA
is:- [AIEEE - 2009]
(2) triatomic
(3) a mixture of monoatomic and diatomic (1) 1076 R (2) 1904 R
(4) monoatomic (3) Zero (4) 276 R
12. A carnot engine, having an efficiency of
17. A diatomic ideal gas is used in a carnot engine as
h = 1/10 as heat engine, is used as a refrigetator.
the working substance. If during the adiabatic
If the work done on the system is 10 J, the amount
expansion part of the cycle the volume of the gas
of energy absorbed from the reservoir at lower
increases from V to 32 V, the efficiency of the
temperature is- [AIEEE - 2007]
engine is :- [AIEEE - 2010]
(1) 99 J (2) 90 J
(1) 0.25 (2) 0.5
(3) 1 J (4) 100 J
(3) 0.75 (4) 0.99
13. When a system is taken from state i to state f
along the path iaf, it is found that Q = 50 cal and 18. A Carnot engine operating between temperatures
W = 20 cal. Along the path ibf Q = 36 cal. W 1
T1 and T2 has efficientcy . When T2 is lowered
along the path ibf is- [AIEEE - 2007] 6
1
by 62 K, its efficiency increases to . Then T1 and
a f 3
T2 are, respectively :- [AIEEE - 2011]

(1) 330 K and 268 K

(2) 310 K and 248 K


i b
(3) 372 K and 310 K
(1) 6 cal (2) 16 cal (4) 372 K and 330 K
(3) 66 cal (4) 14 cal 19. A thermally insulated vessel contains an ideal gas
Directions : Question number 14, 15 and 16 are of molecular mass M and ratio of specific heats g.
based on the following paragraph. It is moving with speed v and is suddenly broght to
Two moles of helium gas are taken over the cycle rest. Assuming no heat is lost to the surroundings,
ABCDA, as shown in the P–T diagram. its temperature increases by :- [AIEEE - 2011]
node06\B0AI-BO\Kota\JEE(MAIN)\Booster Course Sheet\Phy\Eng\Heat, Wave & Modern Physics\02-Thermodynamics.p65

gMv 2
P (1) K
2×10
5 A B 2R

P(Pa)
( g - 1) Mv 2K
(2)
1×10
5

D C
2R
T
300K 500K
T ( g - 1) Mv 2K
(3)
2 ( g + 1) R
14. Assuming the gas to be ideal the work done by the
gas in taking it from A to B is :- [AIEEE - 2009]
(1) 400 R (2) 500 R ( g - 1) Mv 2K
(4)
2 gR
(3) 200 R (4) 300 R

12 E
Thermodynamics
ALLEN
20. 100 g of water is heated from 30°C to 50°C Ignoring 24. One mole of diatomic ideal gas undergoes a cyclic
the slight expansion of the water, the change in its process ABC as shown in figure. The process BC
internal energy is (specific heat of water is is adiabatic. The temperatures at A, B and C are
400 K, 800 K and 600 K respectively. Choose the
4184 J/kg/K) :- [AIEEE - 2011]
correct statement : [JEE(Main)-2014]
(1) 84 kJ (2) 2.1 kJ
B
(3) 4.2 kJ (4) 8.4 kJ 800K

21. A container with insulating walls is divided into two


P
equal parts by a partition fitted with a valve. One
part is filled with an ideal gas at a pressure P and 600K
A C
temperature T, whereas the o ther part is 400K
V
completely evacuated. If the valve is suddenly
opened, the pressure and temperature of the gas (1) The change in internal energy in the process AB
will be :- [AIEEE - 2011] is –350 R.
(2) The change in internal energy in the process BC
P P T is –500R
(1) , T (2) ,
2 2 2
(3) The change in internal energy in whole cyclic
process is 250 R.
T
(3) P, T (4) P, (4) The change in internal energy in the process CA
2
is 700 R.
22. Helium gas goes through a cycle ABCDA (consisting 25. Consider a spherical shell of radius R at temperature
of two isochoric and two isobaric lines) as shown in T. The black body radiation inside it can be
considered as an ideal gas of photons with internal
figure. Efficiency of this cycle is nearly (Assume the
U
gas to be close to ideal gas) :- [AIEEE - 2012] energy per unit volume u = µ T 4 and pressure
V
1æUö
p = ç ÷ . If the shell now undergoes an adiabatic
3èV ø
B C
2P0 expansion the relation between T and R is -
[JEE(Main)-2015]
P0 A D

V0 2V0 1 1
(1) T µ (2) T µ
R R3
(1) 12.5% (2) 15.4% (3) T µ e–R (4) T µ e–3R
26. A solid body of constant heat capacity 1 J°/C is being
node06\B0AI-BO\Kota\JEE(MAIN)\Booster Course Sheet\Phy\Eng\Heat, Wave & Modern Physics\02-Thermodynamics.p65

(3) 9.1% (4) 10.5%


heated by keeping it in contact with reservoirs in
23. A Carnot engine, whose efficiency is 40% takes in two ways - [JEE(Main)-2015]
heat from a source maintained at a temperature of (i) Sequentially keeping in contact with 2 reservoirs
500 K. It is desired to have an engine of efficiency such that each reservoir supplies same amount
60%. Then, the intake temperature for the same of heat.
exhaust (sink) temperature must be :- [AIEEE - 2012] (ii) Sequentially keeping in contact with 8 reservoirs
such that each reservoir supplies same amount
(1) 600 K of heat. In both the cases body is brought from
initial temperature 100°C to final temperature
(2) efficiency of Carnot engine cannot be made
200°C. Entropy change of the body in the two
larger than 50% cases respectively is -
(3) 1200 K (1) ln2, 2ln2 (2) 2ln2, 8ln2
(4) 750 K (3) ln2, 4ln2 (4) ln2, ln2

E 13
JEE ( Main) - Physics
ALLEN
27. Consider an ideal gas confined in an isolated closed 29. An ideal gas undergoes a quasi static, reversible
chamber. As the gas undergoes an adiabatic process in which its molar heat capacity C remains
expansion, the average time of collision between
constant. If during this process the relation of pressure
molecules increases as Vq, where V is the volume
P and volume V is given by PVn = constant, then n
æ Cp ö
of the gas. The value of q is :- ç g = ÷ is given by (Here CP and CV are molar specific heat
è Cv ø
at constant pressure and constant volume,
[JEE(Main)-2015]
respectively) :- [JEE(Main)-2016]

g +1 g -1 3g + 5 3g - 5 C - CV CP
(1) (2) (3) (4)
2 2 6 6 (1) n = (2) n =
C - CP CV
28. 'n' moles of an ideal gas undergoes a process
C – CP CP – C
A ® B as shown in the figure. The maximum (3) n = (4) n =
temperature of the gas during the process will C – CV C - CV
be : [JEE(Main)-2016]
30. Two moles of an ideal monoatomic gas occupies a
volume V at 27°C. The gas expands adiabatically
P to a volume 2V. Calculate (a) the final temperature
2P0 A of the gas and (b) change in its internal energy.
[JEE-Main 2018]
P0 B
(1) (a) 195 K (b) –2.7 kJ
V
V0 2V0
(2) (a) 189 K (b) –2.7 kJ

(3) (a) 195 K (b) 2.7 kJ


9 P0 V0 9 P0 V0 3P0 V0 9 P0 V0 (4) (a) 189 K (b) 2.7 kJ
(1) (2) (3) (4)
nR 4nR 2nR 2nR

node06\B0AI-BO\Kota\JEE(MAIN)\Booster Course Sheet\Phy\Eng\Heat, Wave & Modern Physics\02-Thermodynamics.p65

PREVIOUS YEARS QUESTIONS ANSWER KEY Exercise-II


Que. 1 2 3 4 5 6 7 8 9 10
Ans. 4 3 3 1 3 2 2 4 3 1
Que. 11 12 13 14 15 16 17 18 19 20
Ans. 1 2 1 1 4 4 3 3 2 4
Que. 21 22 23 24 25 26 27 28 29 30
Ans. 1 2 4 2 1 4 1 2 3 2

14 E
Wave Theory
ALLEN
WAVE THEORY EXERCISE-I
1. A travelling wave pulse is given by 6. A transverse periodic wave on a string with a linear
mass density of 0.200 kg/m is described by the
4
y= following equation y = 0.05 sin (420t – 21.0x)
3x + 48t + 24xt + 2
2 2

where x and y are in metres and t is in seconds. The


where x and y are in metre and t is in second. The
tension in the string is equal to :
velocity of wave is :-
(1) 32 N (2) 42 N (3) 66 N (4) 80 N
(1) 4 m/s (2) 2 m/s
7. The figure shows four progressive wave A,B,C &
(3) 8 m/s (4) 12 m/s D. It can be concluded from the figure that with
2. Let speed of sound waves in hydrogen gas at room respect to wave A :-
temperature is v0. What will be the speed of sound
waves in a room which contains an equimolar mixture
of hydrogen and 'He' at same temperature :-

5 7
(1) v (2) v
7 0 5 0

2
(3) v (4) None (1) the wave C is ahead by a phase angle of
5 0
p/2 & the wave B lags behind by a phase angle
3. The displacement wave in a string is
p/2
y = (3 cm) sin 6.28 (0.5x – 50t)
(2) the wave C lags behind by a phase angle of
where x is in centimetres and t in seconds. The p/2 & the wave B is ahead by a phase angle
velocity and wavelength of the wave is :- of p/2
(1) 2 cm, 100 cms–1 (3) the wave C is ahead by a phase angle of p &
(2) 10 cm, 50 cms–1 the wave B lags behind by the phase angle of p
(3) 20 cm, 2 ms–1 (4) the wave C lags behind by a phase angle of p
(4) 2 m, 100 ms –1
& the wave B is ahead by a phase angle of p
4. A uniform rope of mass 6kg hangs vertically from 8. Under similar conditions of temperature and
a rigid support. A block of mass 2kg is attached to pressure, In which of the following gases the velocity
the free end of the rope. Atransverse pulse of of sound will be largest :-
wavelength 0.06m is produced at the lower end of (1) H2 (2) N2 (3) He (4) CO2
the rope. The wavelength of the pulse when it 9. In the figure the intensity of waves arriving at D from
reaches the top of the rope is :- two coherent sources S1 and S2 is I0. The wavelength
of the wave is l = 4m. Resultant intensity at D will
node06\B0AI-BO\Kota\JEE(MAIN)\Booster Course Sheet\Phy\Eng\Heat, Wave & Modern Physics\03-Wave Theory.p65

(1) 0.012m
be :-
(2) 0.06m
(3) 0.24m
(4) 0.12m
5. The following figure depicts a wave travelling in a
medium. Which pair of particles are in phase :-

(1) 4I0 (2) I0 (3) 2I0 (4) zero


10. When a guitar is sounded with a 440 Hz tuning fork,
a beat frequency of 5 Hz is heard. If the experiment
is repeated with a tuning fork of 437 Hz, the beat
(1) A and D (2) B and F frequency is 8 Hz. The string frequency (in Hz) is :-
(3) C and E (4) B and G (1) 445 (2) 435 (3) 429 (4) 448
E 15
JEE(Main)-Physics
ALLEN
11. Two waves are represented by: y1 = 4 sin 404 pt 17. Equation of a standing wave is generally expressed
as y = 2A sin wt coskx. In the equation, quantity w/
and y2 = 3 sin 400 pt. Then :
k represents :-
(1) beat frequency is 4 Hz and the ratio of
(1) the transverse speed of the particles of the string
maximum to minimum intensity is 49 : 1
(2) the speed of either of the component wave
(2) beat frequency is 2 Hz and the ratio of maximum
to minimum intensity is 49 : 1 (3) the speed of the standing wave
(4) a quantity that is independent of the properties
(3) beat frequency is 2 Hz and the ratio of maximum
of the string.
to minimum intensity is 1 : 49
18. The wave - function for a certain standing wave on
(4) beat frequency is 4 Hz and the ratio of
a string fixed at both ends is
maximum to minimum intensity is 1: 49
y(x,t) = 0.5 sin (0.025px) cos500 t
12. 41 tuning forks are arranged such that every fork
gives 5 beats with the next. The last fork has a where x and y are in centimeters and t is in seconds.
the shortest possible length of the string is :-
frequency that is double of the first. The frequency
of the first fork is :- (1) 126 cm (2) 160 cm (3) 40 cm (4) 80 cm

(1) 200 (2) 400 (3) 205 (4) 210 19. A 2.0m long string with a linear mass density of
5.2 ×10–3 kg m–1 and tension 52N has both of its
13. The frequency of a radar is 780 MHz. The frequency
ends fixed. It vibrates in a standing wave pattern
of the reflected wave from an aeroplane is increased
by 2.6 KHz. The velocity of air plane is :- with four antinodes. Frequency of the vibraion is:-

(1) 0.25 km/s (2) 0.5 km/s (1) 75 Hz (2) 150 Hz (3) 100 Hz (4) 50 Hz

(3) 1.0 km/s (4) 2.0 km/s 20. The length of a sonometer wire is 1.25m and
density 8 × 103 kg/m3. It can bear a stress of 3.2
14. An aluminium rod having a length 100 cm is
×108 N/m2 without exceeding the elastic limit. The
clamped at its middle point and set into longitudinal
fundamental frequency that can be produced in the
vibrations. Let the rod vibrate in its fundamental
wire, is :-
mode. The density of aluminium is 2600 kg/m3 and
its Young's modulus is 7.8 × 1010 N/m2. The (1) 100 Hz (2) 80 Hz (3) 200 Hz (4) 250 Hz
frequency of the sound produced is :- 21. An observer moves towards a stationary source of
(1) 1250 Hz (2) 2740 Hz sound, with a velocity one-fifth of the velocity of
sound. What is the percentage increase in he
(3) 2350 Hz (4) 1685 Hz
apparent frequency :-
15. What is the phase difference between the
(1) 5% (2) 20% (3) zero (4) 0.5%
displacement wave and pressure wave in sound
node06\B0AI-BO\Kota\JEE(MAIN)\Booster Course Sheet\Phy\Eng\Heat, Wave & Modern Physics\03-Wave Theory.p65

wave :- 22. A small source of sound moves on a circle as shown


in the figure and an observer is standing on O. Let
p p
(1) Zero (2) (3) p (4) n1, n2 and n3 be the frequencies heard when the
2 4
source is at A, B and C respectively. Then :-
16. A wire of length 'l' having tension T and radius 'r'
vibrates with fundamental frequency 'f'. Another wire
of the same metal with length 2l having tension 2T
and radius 2r will vibrate with fundamental frequency
:-
(1) f (2) 2f
f f (1) n1 > n2 > n3 (2) n2 > n3 > n1
(3) (4) 2
2 2 2 (3) n1 = n2 > n3 (4) n2 > n1 > n3

16 E
Wave Theory
ALLEN
23. A train is moving with 34 m/s towards a stationary 27. In a string the speed of wave is 10 m/s and its
observer. Train sound their whistle and observer frequancy is 100 Hz . The value of the phase
observers its frequency as f1. Now the speed of train difference at a distance 2.5 cm will be :
is decreases to 17 m/s then observed frequency is
(1) p/2 (2) p/8 (3) 3p/2 (4) 2p
f2. If speed of sound is 340 m/s then the ratio of
f1/f2 is :- 28. The equation of a progressive wave are

(1) 18/19 (2) 1/2 (3) 2 (4) 19/18 é æ x öù


Y = sin ê200p ç t - ÷ ú , where x is in meter and
24. The first resonance length of a resonance tube is 40 ë è 330 øû
cm and the second resonance length is 122 cm. the
f is in second. The frequency and velocity of wave
third resonance length of the tube will be :-
are
(1) 200 cm (2)202 cm (3) 203 cm (4) 204 cm
(1) 100 Hz, 5 m/s
25. Two coherent sources of different intensities send (2) 300 Hz, 100 m/s
waves which interfere. The ratio of the maximum
intensity to the minimum intensity is 25. the (3) 100 Hz, 330 m/s
intensities are in the ratio :- (4) 30 m/s, 5 Hz
(1) 25 : 1 (2) 5 ; 1 (3) 9 : 4 (4) 625 :1 29. A sine wave has an amplitude A and wavelength
l. Let V be wave velocity and v be the maximum
26. Two pulses in a stretched string whose centres are
velocity of a particle in medium then.
initially 8 cm apart are moving towards each other
as shown in the figure. The speed of each pulse is
l
2 cm/s. After 2 seconds, the total energy of the (1) V = n if A =
2p
pulses will be :-
(2) V can not be equal to n

3A
(3) V = n if l =
2p

(4) V= n if A = 2 pl
(1) zero 30. The time period of SHM of a particle is 12 s. The
(2) purely kinetic phase difference between the position at t = 3s and
t = 4s will be :
(3) purely potential
(1) p/4 (2) 3p/5 (3) p/6 (4) p/2
(4) partly kinetic and partly potential
node06\B0AI-BO\Kota\JEE(MAIN)\Booster Course Sheet\Phy\Eng\Heat, Wave & Modern Physics\03-Wave Theory.p65

ANSW ER KEY Exercise-I


Que. 1 2 3 4 5 6 7 8 9 10
Ans. 1 1 1 4 1 4 2 1 3 1
Que. 11 12 13 14 15 16 17 18 19 20
Ans. 2 1 2 2 2 3 2 3 3 2
Que. 21 22 23 24 25 26 27 28 29 30
Ans. 2 2 4 4 3 2 1 3 1 3

E 17
JEE(Main)-Physics
ALLEN
PREVIOUS YEARS' QUESTIONS EXERCISE-II
1. Tube A has both ends open while tube B has one 7. When two tuning forks (fork 1 and fork 2) are
end closed, otherwise they are identical. The ratio sounded simultaneously, 4 beats per second are
of fundamental frequency of tubes A and B is- heard. Now, some tape is attached on the prong
[AIEEE - 2002]
of the fork 2. When the tuning forks are sounded
(1) 1 : 2 (2) 1 : 4
again, 6 beats per second are heard. If the
(3) 2 : 1 (4) 4 : 1
frequency of fork 1 is 200 Hz, then what was the
2. A tuning fork arrangement (pair) produces 4beats/s
original frequency of fork 2 ? [AIEEE - 2005]
with one fork of frequency 288 cps. A little wax is
placed on the unknown fork and it then produces (1) 200 Hz (2) 202 Hz
2 beats/s. The frequency of the unknown fork is- (3) 196 Hz (4) 204 Hz
[AIEEE - 2002] 8. An observer moves towards a stationary source of
(1) 286 cps (2) 292 cps sound, with a velocity one-fifth of the velocity of
(3) 294 cps (4) 288 cps sound. what is the percentage increase in the
3. A wave y = asin(wt – kx) on a string meets with apparent frequency ? [AIEEE - 2005]
another wave producing a node at x = 0. Then the
(1) zero (2) 0.5%
equation of the unknown wave is-
(3) 5% (4) 20%
(1) y = asin(wt + kx) [AIEEE - 2002]
9. A whistle producing sound waves of frequencies
(2) y = – asin(wt + kx)
9500 Hz and above is approaching a stationary
(3) y = asin(wt – kx)
person with speed v ms–1. The velocity of sound in
(4) y = – asin(wt – kx)
4. Length of a string tied to two rigid supports is air is 300 ms–1. If the person can hear frequencies

40 cm. Maximum length (wavelength in cm) of a upto a maximum of 10,000 Hz, the maximum
stationary wave produced on it, is- [AIEEE - 2002] value of v upto which he can hear the whistle is-
(1) 20 (2) 80 [AIEEE - 2006]
(3) 40 (4) 120
(1) 15 2 ms–1 (2) 15/ 2 ms–1
5. The displacement y of a wave travelling in the
x-direction is given by (3) 15 ms–1 (4) 30 ms–1
10. A sound absorber attenuates the sound level by
p
y = 10–4 sin æç 600 t - 2x + ö÷ metre,
è 3ø 20 dB. The intensity decreases by a factor of-

where, x is expressed in metres and t in seconds. [AIEEE - 2007]


node06\B0AI-BO\Kota\JEE(MAIN)\Booster Course Sheet\Phy\Eng\Heat, Wave & Modern Physics\03-Wave Theory.p65

The speed of the wave-motion, in ms–1 is- (1) 1000 (2) 10000
[AIEEE - 2003] (3) 10 (4) 100
(1) 300 (2) 600 11. While measuring the speed of sound by performing
(3) 1200 (4) 200 a resonance column experiment, a student gets the
6. A tuning fork of known frequency 256 Hz makes first resonance condition at column length of
5 beats per second with the vibrating string of a
18 cm duri ng winte r. Repe atio n th e sa me
piano. The beat frequency decreases to 2 beats
experiment during sumer, student measures the
per second when the tension in the piano string is
column length to be x cm for the second resonance.
slightly increased. The frequency of the piano string
before increasing the tension was- [AIEEE - 2003]
Then [AIEEE - 2008]

(1) (256 + 2) Hz (2) (256 – 2) Hz (1) 18 > x (2) x > 54


(3) (256 – 5) Hz (4) (256 + 5) Hz (3) 54 > x > 36 (4) 36 > x > 18

18 E
Wave Theory
ALLEN
12. A wave travelling along the x-axis is described by 16. The transverse displacement y(x, t) of a wave on a
string is given by
the equation y (x, t ) = 0.005 cos ( ax - bt ). If the

y ( x, t ) = e
(
– ax2 +bt2 +2 abxt )
wavelength and the time period of the wave in
0.08m and 2.0 s respectively then a and b in This represents a :- [AIEEE - 2011]
appropriate units are [AIEEE - 2008] (1) standing wave of frequency b
(1) a = 25.00p, b = p 1
(2) standing wave of frequency
b
0.08 2.0 a
(2) a = , b= (3) wave moving in +x directionwith speed
p p b
b
0.04 1.0 (4) wave moving in –x direction with speed
(3) a = ,b = a
p p
17. Statement-1 :
Two longitudinal waves given by equations :
p
(4) a = 12.50 p , b = y1(x, t) = 2a sin (wt–kx) and y2(x, t) = a sin(2wt – 2kx)
2.0
will have equal intensity.
13. Three sound waves of equal amplitudes have fre- Statement-1:
quencies (n–1), n, (n+1). They superpose to give Intensity of waves of given frequency in same
medium is proportional to square of amplitude only.
beats. The number of beats produced per second
[AIEEE - 2011]
will be :- [AIEEE - 2009]
(1) Statement-1 is false, statement-2 is true.
(1) 2 (2) 1
(2) Statement-1 is true, statement-2 is false.
(3) 4 (4) 3
(3) Statement-1 is true, statement-2 true; statement-2
14. A motor cycle starts from rest and accelerates along is the correct explanation of statement-1
a straight path at 2 m/s2. At the starting point of (4) Statement-1 is true, statement-2 is true;
the motor cycle there is a stationary electric siren. statement -2 is not correct explanation of

How far has the motor cycle gone when the driver statement-1.
18. A travelling wave represented by y = A sin(wt–kx)
hears the frequency of the siren at 94% of its value
is superimposed on another wave represented by
when the motor cycle was at rest ?
y = A sin (wt + kx). The resultant is :-
(Speed of sound = 330 ms–1) :- [AIEEE - 2009] [AIEEE - 2011]
node06\B0AI-BO\Kota\JEE(MAIN)\Booster Course Sheet\Phy\Eng\Heat, Wave & Modern Physics\03-Wave Theory.p65

(1) 147 m (2) 196 m (1) A stan ding wave h aving no des at
(3) 49 m (4) 98 m
æ 1ö l
x = n+ ,n = 0,1,2
15. The equation of a wave on a string of linear è 2ø 2
mass density 0.04 kg m –1 is given by

(2) A wave travelling along + x direction


é æ t x öù
y = 0.02(m) sin 2p ç
êë è 0.04(s) – 0.50(m) ÷ø úû .
(3) A wave travelling along –x direction

The tension in the string is : [AIEEE - 2010] nl


(4) A standing wave having nodes at x = ;
2
(1) 6.25 N (2) 4.0 N
(3) 12.5 N (4) 0.5 N n = 0, 1, 2

E 19
JEE(Main)-Physics
ALLEN
19. A cylindrical tube, open at both ends, has a 23. A uniform string of length 20m is suspended from
fundamental frequency, f, in air. The tube is dipped a rigid support. A short wave pulse is introduced at
vertically in water so that half of it is in water. The its lowest end. It starts moving up the string. The
fundamental frequency of the air-column is now :- time taken to reach the support is :-[JEE(Main) 2016]
[AIEEE - 2012] (take g = 10 ms–2)
(1) 2f (2) f (1) 2s (2) 2p 2 s
(3) f/2 (4) 3f/4
(3) 2s (4) 2 2 s
20. A sonometer wire of length 1.5m is made of steel.
24. A pipe open at both ends has a fundamental
The tension in it produces an elastic strain of 1%.
frequency f in air. The pipe is dipped vertically in
What is the fundamental frequency of steel if density
water so that half of it is in water. The fundamental
and elasticity of steel are 7.7 × 103 kg/m3 and
frequencty of the air column is now :-
2.2 ×1011 N/m2 respectively ? [JEE(Main) 2013]
[JEE(Main) 2016]
(1) 188.5 Hz (2) 178.2 Hz
(3) 200.5 Hz (4) 770 Hz f 3f
(1) f (2) (3) (4) 2f
2 4
21. A pipe of length 85 cm is closed from one end.
25. An observer is moving with half the speed of light
Find the number of possible natural oscillations of
towards a stationary microwave source emitting
air column in the pipe whose frequencies lie below
waves at frequency 10 GHz. What is the frequency
1250 Hz. The velocity of sound in air is 340 m/s.
of the microwave measured by the observer?
[JEE(Main) 2014]
(speed of light = 3 × 108 ms–1)
(1) 6 (2) 4
[JEE(Main) 2017]
(3) 12 (4) 8
(1) 17.3 GHz (2) 15.3 GHz
22. A train is moving on a straight track with speed (3) 10.1 GHz (4) 12.1 GHz
20 ms–1. It is blowing its whistle at the frequency of 26. A granite rod of 60 cm length is clamped at its
1000 Hz. The percentage change in the frequency middle point and is set into longitudinal vibrations.
heard by a person standing near the track as the The density of granite is 2.7 × 103 kg/m3 and its
train passes him is (speed of sound = 320 ms–1) close Young's modulus is 9.27 ×1010 Pa. What will be the
to:- [JEE(Main) 2015]
fundamental frequency of the longitudinal vibrations?
(1) 18% (2) 24% [JEE(Main) 2018]
(3) 6% (4) 12%
(1) 2.5 kHz (2) 10 kHz
(3) 7.5 kHz (4) 5 kHz
node06\B0AI-BO\Kota\JEE(MAIN)\Booster Course Sheet\Phy\Eng\Heat, Wave & Modern Physics\03-Wave Theory.p65

PREVIOUS YEARS QUESTIONS ANSWER KEY Exercise-II


Que. 1 2 3 4 5 6 7 8 9 10
Ans. 3 2 2 2 1 3 3 4 3 4
Que. 11 12 13 14 15 16 17 18 19 20
Ans. 2 1 1 4 1 4 2 1 2 2
Que. 21 22 23 24 25 26
Ans. 1 4 4 1 1 4

20 E
Ray Optics
ALLEN
RAY OPTICS EXERCISE-I
1. The distance of an object from a spherical mirror is 7. Figure shows graph of deviation d versus angle of
equal to the focal length of the mirror. Then the incidence for a light ray striking a prism. Angle of
image: prism is :-
(1) must be at infinity
(2) may be at infinity d
30º
(3) may be at the focus
(4) none
2. An object is placed in front of a spherical mirror
whose 2 times magnified image is formed on screen. 15º 60º
Then choose CORRECT option :-
(1) 30º (2) 45º
(1) Mirror is concave m = +2
(3) 60º (4) 75º
(2) Mirror is concave m = –2
(3) Mirror is convex m = +2 8. There is a prism with refractive index equal to 2
(4) Mirror is convex m = –2
and the refracting angle equal to 30°. One of the
3. When a ray of light of frequency 6 ×1014 Hz travels
from water of refractive index 4/3 to the glass of refracting surface of the prism is polished. A beam
refractive index 8/5, its :- of monochromatic light will retrace its path if its
(1) frequency decreases to 5/6 of its initial value angle of incidence over the first refracting surface
(2) speed decreases to 5/6 of its initial value of the prism is :-
(3) wavelength decreases to 6/5 of its initial value (1) 0° (2) 30°
(4) speed increases to 6/5 of its initial value (3) 45° (4) 60°
4. A ray of light is incident at the glass–water interface
9. The refractive index for the material of a 60° prism
at an angle i, it emerges finally parallel to the
is 1.50. Then the angle of incidence for minimum
surface of water, then the value of µ g would be :–
deviation is nearly.
Air
r
µw=4/3 2 3
( sin 42 ° » and sin 49 ° » )
Water
r 3 4
Glass
i (1) 30° (2) 49°
(3) 38° (4) 28°
(1) (4/3) sin i (2) 1/sin i
(3) 4/3 (4) 1 10. A ray of light is incident on an equilateral glass prism
placed on a horizontal table. For minimum
node06\B0AH-AI\Kota\JEE(MAIN)\Crash Course Sheet\Phy\Eng\Heat, Wave & Modern Physics\04-Ray Optics.p65

5. A ray of light is incident at 60° on a prism of


refracting angle 30°. The emerging ray is at an deviation which of the following is true ?
angle 30° with the incident ray. The value of
refractive index of the prism is :-
Q R
2 S
3 3 P
(1) (2) (3) 3 (4)
4 2 3
(1) PQ is horizontal
6. The refracting angle of the prism is 60° and
minimum deviation of 30°, then the angle of (2) QR is horizontal
incidence is:- (3) RS is horizontal
(1) 30° (2) 45°
(4) Either PQ or RS is horizontal
(3) 25° (4) 60°

E 21
JEE ( Main) - Physics
ALLEN
11. One end of a glass rod of refractive index n = 1.5 15. A ray gets succesively reflected from two mirrors
is a spherical inclined at an angle of 40°. If the angle of incidence
on the first mirror is 30° then the net deviation of
this ray is
µ=1.5
(1) 40° (2) 280°
P O Q
(3) 80° (4) 240°
16. A small object's 10 cm in front of a plane mirror. A
man stands 30 cm from the mirror, behind the
surface of radius of curvature R. The centre of the
object and looks at the object's image. He should
spherical surface lies inside the glass. A point object
focus his eyes to see the image at a distance
placed in air on the axis of the rod at the point P
has its real image inside glass at the point Q (see (1) 25 cm (2) 35 cm
fig.). A line joining the points P and Q cuts the surface (3) 45 cm (4) 40 cm
at O such that OP = 2OQ. The distance PO is :-
17. An object 5 cm tall is placed 1 m from a concave
(1) 8 R (2) 7 R spherical mirror which has a radius of curvature of
(3) 2 R (4) None of these 20 cm. The size of the image is :–
12. A point object is placed at the centre of a glass (1) 0.11 cm (2) 0.50 cm
sphere of radius 6 cm and refractive index 1.5.
The distance of the virtual image from the surface (3) 0.55 cm (4) 0.60 cm
of the sphere is :- 18. The focal length of a concave mirror is 12 cm.
(1) 2 cm (2) 4 cm Where should an object of length 4 cm be placed,
(3) 6 cm (4) 12 cm so that a real image of 1 cm length is formed ?

13. A concave lens of glass, refractive index 1.5, has (1) 48 cm (2) 3 cm
both surfaces of same radius of curvature R. On (3) 60 cm (4) 15 cm
immersion in a medium of refractive index 1.75, it
19. The wavelength of light in two liquids 'x' and 'y' is
will behave as a :-
3500 Å and 7000 Å. Then the critical angle of x
(1) convergent lens of focal length 3.5R relative to y will be
(2) convergent lens of focal length 3.0 R. (1) 60° (2) 45°
(3) divergent lens of focal length 3.5 R (3) 30° (4) 15°
(4) divergent lens of focal length 3.0 R 20. A bird in air looks a fish vertically below it and
14. An equiconvex lens of refractive index µ and radius inside water ; h1 is the height of the bird above the
of curvature R has its one surface silvered. A point surface of water and h2 the depth of the fish below
source O is placed before the silvered lens so that the surface of water. If refractive index of water
its image is coincident with it, the distance of the with respect to air be µ, then the distance of the
node06\B0AH-AI\Kota\JEE(MAIN)\Crash Course Sheet\Phy\Eng\Heat, Wave & Modern Physics\04-Ray Optics.p65

object from the lens is :- fish as observed by the bird is

h2
\\\\

(2) h1 +
\\\\\\\\\\\\\\\\\\\\\\\

(1) h1 + h2
µ
O
(3) µh1 + h2 (4) µh1 + µh2
21. An air bubble in a glass slab (µ = 1.5) is 6 cm deep
R 2R as viewed from one face and 4 cm deep as viewed
( )
(1) m - 1
( )
(2) m - 1
from the other face. The thickness of the glass slab
is
R 2R (1) 6.67 cm (2) 10 cm
(3)
( 2m - 1) (4)
( 2m - 1) (3) 15 cm (4) Data is incomplete

22 E
Ray Optics
ALLEN
22. The minimum distance between an object and its 27. A ray of light making an angle 10° with the horizontal
real image formed by a convex lens is is incident on a plane mirror an angle q with the
horizontal. What should be the value of q so that
(2) 2f (2) 4f
the reflected ray goes vertically upwards ?
(3) f (4) zero
(1) 20° (2) 30°
23. A point object is placed at a distance of 20 cm (3) 40° (4) 45°
form a thin plano-convex lens of focal length 28. What should be the maximum acceptance angle
15 cm. If the plane surface is silvered, the image at the air–core interface of an optical fibre if n1
will from at and n2 are the refractive indices of the core and
A the cladding, respectively

(1) sin–1(n2/n1) (2) sin–1 n12 – n 22


O

LM OP LM OP
20cm

–1 n 2 –1 n1
(3) tan n (4) tan n
B
N 1 Q N 2 Q
(1) 60 cm from left of AB
(2) 30 cm from left of AB 29. A telescope consisting of an objective of focal length
(3) 12 cm from left of AB 60 cm and an eyepiece of focal length 5 cm is
(4) 60 cm from right of AB focussed to a distant object in such a way that parallel
24. A convex lens is made up of three different materials
as shown in the figure. For a point object placed on rays emerge from the eye piece. If the object
its axis, the number of images formed are subtends an angle of 2° at the objective, then find
the angular width of the image.
(1) 24° (2) 26°
(3) 21° (4) 20°
30. A prism having an apex angle of 4° and refractive
index of 1.50 is located in front of a vertical plane
(1) 1 (2) 3 (3) 4 (4) 5 mirror as shown. A horizontal ray of light is incident
25. If tube length of astronomical telescope is 105 cm on the prism. The total angle through which the ray
and magnifying power is 20 for normal setting. is deviated is :-
Calculate the focal length of objective :–
(1) 100 cm (2) 10 cm P
(3) 20 cm (4) 25 cm
26. A point object is moving on the principal axis of a
concave mirror of focal length 24 cm, towards the
mirror. When it is at a distance of 60 cm from the
mirror, its velocity is 9 cm/sec. What is the velocity
of the image at that instant M
node06\B0AH-AI\Kota\JEE(MAIN)\Crash Course Sheet\Phy\Eng\Heat, Wave & Modern Physics\04-Ray Optics.p65

(1) 5 cm/sec towards the mirror


(2) 4 cm/sec towards the mirror
(1) 4° clockwise (2) 178° clockwise
(3) 4 cm/sec away from mirror
(3) 2° clockwise (4) 8° clockwise
(4) 9 cm/sec away from mirror

ANSW ER KEY E xercis e-I


Que. 1 2 3 4 5 6 7 8 9 10
Ans. 2 2 2 2 3 2 2 3 2 2
Que. 11 12 13 14 15 16 17 18 19 20
Ans. 1 3 1 3 2 4 3 3 3 2
Que. 21 22 23 24 25 26 27 28 29 30
Ans. 3 2 3 2 1 3 3 2 1 2

E 23
JEE ( Main) - Physics
ALLEN
PREVIOUS YEARS' QUESTIONS EXERCISE-II
1. Wavelength of light used in a optical instrument 8. A plano-convex lens of refractive index 1.5 and
are l1 = 4000 Å and l2 = 5000 Å, then ratio of radius of curvature 30 cm is silvered at the curved
their respective resolving powers (corresponding surface. Now, this lens has been used to form the
to l1 and l2) is- [AIEEE - 2002]
image of an object. At what distance from this lens,
(1) 16 : 25 (2) 9 : 1
an object be placed in order to have a real image
(3) 4 : 5 (4) 5 : 4
of the size of the object? [AIEEE-2004]
2. An astronomical telescope has a large aperture
to- [AIEEE - 2002]
(1) 20 cm (2) 30 cm

(1) reduced spherical aberration (3) 60 cm (4) 80 cm


(2) have high resolution 9. A fish looking up through the water sees the outside
(3) increase span of observation world, contained in a circular horizon. If the
(4) have low dispersion 4
3. If two mirrors are kept at 60° to each other, then refractive index of water is and the fish is
3
the number of images formed by them is- 12 cm below the water surface, the radius of this
[AIEEE- 2002] circle in cm is- [AIEEE-2005]
(1) 5 (2) 6 (3) 7 (4) 8
36
4. Which of the following is used in optical fibres? (1) 36 7 (2)
(1) Total internal reflection [AIEEE - 2002] 7
(2) Scattering
(3) 36 5 (4) 4 5
(3) Diffraction
(4) Refraction 10. A thin glass (refractive index 1.5) lens has optical
5. The image formed by an objective of a compound power of – 5D in air. Its optical power in a liquid
microscope is- [AIEEE - 2003] medium with refractive index 1.6 will be-
(1) virtual and diminished (2) real and diminished [AIEEE-2005]
(3) real and enlarged (4) virtual and enlarged (1) 1 D (2) – 1D
6. To get three images of a single object, one should
(3) 25 D (4) – 25 D
have two plane mirrors at an angle of-
11. Two point white dots are 1 mm apart on a black
(1) 60° (2) 90° [AIEEE - 2003]
paper. They are viewed by eye of pupil diameter
(3) 120° (4) 30°
3 mm. Approximately, what is the maximum
7. A light ray is incident perpendicular to one face of
a 90° prism and is totally internally reflected at the distance at which these dots can be resolved by the
glass-air interface. If the angle of reflection is 45°, eye ? [Take wavelength of light = 500 nm]
we conclude that the refractive index n : [AIEEE-2005]
[AIEEE-2004] (1) 5 m (2) 1 m
node06\B0AH-AI\Kota\JEE(MAIN)\Crash Course Sheet\Phy\Eng\Heat, Wave & Modern Physics\04-Ray Optics.p65

(3) 6 m (4) 3 m
12. The refractive index of glass is 1.520 for red light
and 1.525 for blue light. Let D1 and D2 be angles
45°
of minimum deviation for red and blue light
45° respectively in a prism of this glass. Then-
(1) D1 < D2 [AIEEE-2006]

1 (2) D1 = D2
(1) n < (2) n > 2
2 (3) D 1 can be less than or greater than D 2
depending upon the angle of prism
1
(3) n > (4) n < 2 (4) D1 > D2
2

24 E
Ray Optics
ALLEN
13. Two lenses of power – 15D and +5D are in contact 17. The focal length of a thin biconvex lens is 20 cm.
with each other. The focal length of the combination When an object is moved from a distance of
is- [AIEEE-2007] 25 cm in front of it to 50 cm, the magnification of
(1) – 20 cm (2) – 10 cm
m25
(3) + 20 cm (4) + 10 cm its image changes from m25 to m50. The ratio
m50
14. A stuydent measures the focal length of a convex
lens by putting an object pin at a distance ‘u’ from is [IIT-JEE 2010]

the lens and measuring the distance ‘v’ of the image (1) 2 (2) 4
pin. The graph between ‘u’ and ‘v’ plotted by the (3) 6 (4) 8
student should look like- [AIEEE - 2008]
18. Image of an object approaching a convex mirror of
v(cm) v(cm) radius of curvature 20 m along its optical axis is

25 50
(1) (2) observed to move from m to m in
3 7
O u(cm) O u(cm)
30 seconds. What is the speed of the object in km
v(cm) v(cm) per hour? [IIT-JEE 2010]

(1) 3 (2) 6
(3) (4)
(3) 4 (4) 8
O u(cm) O u(cm)

15. A transparent solid cyclindrical rod has a refractive æ 5ö


19. A large glass slab çè m = ÷ø of thickness 8 cm is
3
2
index of . It is surrounded by air. A light ray is placed over a point source of light on a plane
3
surface. It is seen that light emerges out of the top
incident at the mid-point of one end of the rod as surface of the slab from a circular area of radius
shown in the figure. [AIEEE - 2009] R cm. What is the value of R? [IIT-JEE 2010]

(1) 2 (2) 3
q (3) 5 (4) 6
20. A car is fitted with a convex side-view mirror of
The incident angle q for which the light ray grazes focal length 20 cm. A second car 2.8 m behind the
along the wall of the rod is :- first car is overtaking the first car at a relative speed
of 15 m/s. The speed of the image of the second
æ 2 ö æ 1 ö car as seen in the mirror of the first one
(1) sin–1 ç ÷ (2) sin–1 ç ÷
è 3ø è 3ø is :- [AIEEE - 2011]
node06\B0AH-AI\Kota\JEE(MAIN)\Crash Course Sheet\Phy\Eng\Heat, Wave & Modern Physics\04-Ray Optics.p65

(1) 10 m/s (2) 15 m/s


æ1ö æ 3ö
(3) sin–1 ç ÷ (4) sin–1 çç 2 ÷÷
è2ø è ø 1 1
(3) m/s (4) m/s
10 15
16. A biconvex lens of focal length 15 cm is in front of
a plane mirror. The distance between the lens and 21. When monochromatic red light is used instead of
the mirror is 10 cm. A small object is kept at a blue light in a convex lens, its focal length
distance of 30 cm from the lens. The final image is will :- [AIEEE - 2011]

[IIT-JEE 2010] (1) Does not depend on colour of light


(1) virtual and at a distance of 16 cm from the mirror (2) Increase
(2) real and at a distance of 16 cm from the mirror (3) Decrease
(3) virtual and at a distance of 20 cm from the mirror
(4) Remain same
(4) real and at a distance of 20 cm from the mirror

E 25
JEE ( Main) - Physics
ALLEN
22. A beaker contains water up to a height h1 and 25. Which of the following processes play a part in the
kerosene of height h2 above watger so that the total formation of a rainbow ? [AIEEE - 2012 (Online)]
height of (water + kerosene) is (h1 + h2). Refractive (a) refraction
index of water is µ 1 and that of kerosene is µ 2. The
(b) total internal reflection
apparent shift in the position of the bottom of the
(c) dispersion
beaker when viewed from above is :-
[AIEEE - 2011] (d) interference
(1) a, b and c (2) a and b
æ 1ö æ 1ö
(1) ç 1 - ÷ h2 + ç 1 - ÷ h1 (3) c and d (4) a, b and d
è µ1 ø è µ2 ø
26. A glass prism of refractive index 1.5 is immersed
in water (refractive index 4/3) as shown in the figure.
æ 1ö æ 1ö
(2) ç 1 + ÷ h1 - ç 1 + ÷ h2 A light beam incident normally on the face AB is
è µ1 ø è µ2 ø
totally reflected to reach the face BC, if:-
[AIEEE - 2012 (Online)]
æ 1ö æ 1ö
(3) ç 1 - µ ÷ h1 + ç 1 - µ ÷ h2
è 1 ø è 2ø B A
q

æ 1ö æ 1ö
(4) ç 1 + ÷ h2 - ç 1 + ÷ h1
è µ1 ø è µ2 ø
C
23. An object 2.4 m in front of a lens forms a sharp
image on a film 12 cm behind the lens. A glass
plate 1 cm thick, of refractive index 1.50 is
interposed between lens and film with its plane faces 5 1
parallel to film. At what distance (from lens) (1) sin q > (2) sin q >
9 3
should object be shifted to be in sharp focus on
film? [AIEEE- 2012] 2 8
(1) 5.6 m (2) 7.2 m (3) sin q > (4) sin q >
3 9
(3) 2.4 m (4) 3.2 m
24. A beam of light consisting of red, green and blue 27. A biconvex lens is formed with two thin
colours is incident on a right-angled prism on face plano-convex lenses as shown in the figure,
AB. The refractive indices of the material for the Refractive index n of the first lens is 1.5 and that
above red, green and blue wavelength are 1.39, of the second lens is 1.2. Both the curved surfaces
1.44 and 1.47 respectively. A person looking on are of the same radius of curvature R = 14 cm.
surface AC of the prism will see : For this biconvex lens, for an object distance of
40 cm, the image distance will be :-
node06\B0AH-AI\Kota\JEE(MAIN)\Crash Course Sheet\Phy\Eng\Heat, Wave & Modern Physics\04-Ray Optics.p65

A
[IIT - 2012]

n=1.5 n=1.2
45°
C
B

[AIEEE - 2012 (Online)]


(1) red and green colours
(2) No light R=14cm
(3) green and blue colours
(4) red colour only (1) –280.0 cm (2) 40.0 cm
(3) 21.5 cm (4) 13.3 cm

26 E
Ray Optics
ALLEN
Paragraph for Questions 28 and 29 29. Choose the correct statement. [IIT-JEE 2012]

Most materials have the refractive index, n >1. (1) The speed of light in the meta-material is
So, when a light ray from air enters a naturally v = c|n|
occurrin g ma terial, t hen by Sn ell' s la w, (2) The speed of light in the meta-material is
c
sin q1 n2 v=
= , it is understood that the refracted ray n
sin q2 n1 (3) The speed of light in the meta-material is v = c.

bends towards the normal. But it never emerges (4) The wavelength of the light in the meta-material
on the same side of the normal as the incident ray. (lm) is given by lm = lair|n|, where lair is the
According to electromagnetism, the refractive wavelength of the light in air.
index of the medium is given by the relation, 30. Diameter of a plano-convex lens is 6cm and
æcö thickness at the centre is 3 mm. If speed of light in
n = ç ÷ = ± e r mr , where c is the speed of
èvø material of lens is 2 × 108 m/s, the focal length of
electromagnetic waves in vacuum, v its speed in
the lens is : [JEE(Main)-2013]
the medium, er and mr are the relative permittivity
(1) 15 cm (2) 20 cm
and permeability of the medium respectively.
(3) 30 cm (4) 10 cm
In normal materials, both er and mr are positive,
implying positive n for the medium. When both er 31. The graph between angle of deviation (d) and angle

and mr are negative, one must choose the negative of incidence (i) for a triangular prism is represented

root of n. Such negative refractive index materials by :- [JEE(Main)-2013]

can now be artificially prepared and are called


meta-materials. They exhibit significantly different
optical behaviour, without violating any physical
laws. Since n is negative, it results in a change in (1) (2)
the direction of propagation of the refracted light.
However, similar to normal materials, the frequency
of light remains unchanged upon refraction even

in meta-materials.

28. For light incident from air on a meta-material, the


(3) (4)
appropriate ray diagram is [IIT-JEE 2012]
node06\B0AH-AI\Kota\JEE(MAIN)\Crash Course Sheet\Phy\Eng\Heat, Wave & Modern Physics\04-Ray Optics.p65

q1 q1 32. The image of an object, formed by a plano-convex


Air Air
(1) (2) lens at a distance of 8 m behind the lens, is real
Meta-material Meta-material q2
q2
and is one-third the size of the object. The
2
wavelength of light inside the lens is times the
3
wavelength in free space. The radius of the curved
q1 q1
Air Air surface of the lens is :- [IIT-JEE 2013]
(3) Meta-material
(4) Meta-material
q2 q2 (1) 1 m (2) 2 m

(3) 3 m (4) 6 m

E 27
JEE ( Main) - Physics
ALLEN
35. Assuming human pupil to have a radius of 0.25cm
æ 3ö
33. A thin convex lens made from crown glass ç µ = ÷ and a comfortable viewing distance of 25cm, the
è 2ø
minimum separation between two objects that
has focal length ƒ. When it is measured in two
human eye can resolve at 500 nm wavelength is :-
4
different liquids having refractive indices and [JEE(Main)-2015]
3
(1) 100 µm (2) 300 µm (3) 1 µm (4) 30 µm
5
, it has the focal length ƒ1 and ƒ2 respectively. 36. A diverging lens with magnitude of focal length
3
25 cm is placed at a distance of 15 cm from a
The correct relation between the focal lengths is : converging lens of magnitude of focal length
[JEE(Main)-2014] 20 cm. A beam of parallel light falls on the diverging
(1) ƒ2 > ƒ and ƒ1 becomes negative lens. The final image formed is :[JEE(Main)-2017]
(2) ƒ1 and ƒ2 both become negative (1) real and at a distance of 40 cm from the
(3) ƒ1 = ƒ2 < ƒ divergent lens
(4) ƒ1 > ƒ and ƒ2 become negative
(2) real and at a distance of 6 cm from the
34. A green light is incident from the water to the air - convergent lens
water interface at the critical angle (q). Select the
(3) real and at a distance of 40 cm from convergent
correct statement :- [JEE(Main)-2014]
lens
(1) The spectrum of visible light whose frequency is
more than that of green light will come out to (4) virtual and at a distance of 40 cm from
the air medium. convergent lens.

(2) The entire spectrum of visible light will come


out of the water at various angles to the normal
(3) The entire spectrum of visible light will come
out of the water at an angle of 90° to the normal.
(4) The spectrum of visible light whose frequency is
less than that of green light will come out to the
air medium.

node06\B0AH-AI\Kota\JEE(MAIN)\Crash Course Sheet\Phy\Eng\Heat, Wave & Modern Physics\04-Ray Optics.p65

PREVIOUS YEARS QUESTIONS ANSWER KEY Exercise-II


Que. 1 2 3 4 5 6 7 8 9 10
Ans. 4 2 1 1 3 2 2 1 2 1
Que. 11 12 13 14 15 16 17 18 19 20
Ans. 1 1 2 3 2 2 3 1 4 4
Que. 21 22 23 24 25 26 27 28 29 30
Ans. 2 3 1 4 1 4 2 3 2 3
Que. 31 32 33 34 35 36
Ans. 3 3 4 4 4 3

28 E
Wave Optics
ALLEN
WAVE OPTICS EXERCISE-I
1. In the Young's double slit experiment, the 5. If a thin mica sheet of thickness t and refractive index
µ = (5/3) is placed in the path of one of the
interference pattern is found to have an intensity interfering beams as shown in figure, then the
ratio between bright and dark fringes, as 9. This displacement of the fringe system is :-
implies that :- S1 d t P

(1) the intensities at the screen due to the two slits


2d
are 5 units and 1 unit respectively

(2) the intensities at the screen due to the two slits S2 D


are 4 units and 1 unit respectively
Dt Dt
(1) (2)
(3) the amplitude ratio of sources is 4 3d 5d

(4) the amplitude ratio of sources is 2 Dt 2Dt


(3) (4)
4d 5d
2. In Young's double slit interference experiment, the
distance between two sources is 0.1 mm. The 6. A thin sheet of glass (µ = 1.5) of thickness 6 microns
introduced in the path of one of interfering beams
distance of the screen from the sources is 20 cms. in a double slit experiment shifts the central fringe
Wavelength of light used is 5460 Å. Then the to a position previously occupied by fifth bright
angular position of the first dark fringe is :- fringe. Then the wavelength of light used is :-
(1) 6000 Å (2) 3000 Å
(1) 0.08º (2) 0.16°
(3) 4500 Å (4) 7500 Å
(3) 0.20° (4) 0.32° 7. When light wave travelling in air suffers reflection
at the interface between air and glass, the change
3. White light is used to illuminate the two slits in a
of phase of the reflected wave is equal to :-
Young's double slit experiment. The separation
(1) zero (2) p/2
between slits is b and the screen is at a distance d (3) p (4) 2p
(>>b) from the slits. At a point on the screen directly 8. In Young's double slit experiment, the intensity of
infront of one of the slits, certain wavelengths are light at a point on the screen where the path
difference is l is l0. The intensity of light at a point
missing. Some of these missing wavelengths are :-
l
2 2
where the path difference becomes is:-
(1) l = b /2d (2) l = 2b /d 3
node06\B0AI-BO\Kota\JEE(MAIN)\Booster Course Sheet\Phy\Eng\Heat, Wave & Modern Physics\05-Wave Optics.p65

(3) l = b2/3d (4) l = 2b2/3d l0 l0 l0


(1) l0 (2) (3) (4)
4 3 2
4. In two separate setup of the Young's double slit
9. If two coherent sources are placed at a distance 3l
experiment, fringes of equal width are observed from each other symmetric to the centre of the circle
when lights of wavelengths in the ratio 1 : 2 are used. shown in the figure, then number of maxima shown
If the ratio of the slit separation in the two cases is on the screen placed along the circumference is :-

2 : 1, the ratio of the distances between the plane


3l
of the slits and the screen, in the two setup is :-
S1 S2
(1) 4 : 1 (2) 1 : 1

(3) 1 : 4 (4) 2 : 1
(1) 16 (2) 12 (3) 8 (4) 4

E 29
JEE(Main)-Physics
ALLEN
10. Light of wavelength l is incident on a slit of width 15. In an interference pattern the (n+4)th blue bright
d. The resulting diffraction pattern is observed on fringe and nth red bright fringe are formed at the
a screen at a distance D. The linear width of the same spot. If red and blue light have the wavelength
central maxima equal to width of the slit if D equals of 7800 Å and 5200 Å then value of n should be :

(1) d/l (2) 2l/d (1) 2 (2) 4 (3) 6 (4) 8

(3) d2/2l (4) 2l2/d 16. The intensity of the central fringe obtained in the
interference pattern due to two indentical slit
11. Light of wavelength 6328 Å is incident normally
sources is I. When one of the slits is closed then the
on a slit having a width of 0.2 mm. The width of
intensity at the same point is I0. Then the correct
the central maximum measured from minimum to
relation between I and I0 is :
minimum of diffraction pattern on a screen
9.0 metres away from slit will be about :- (1) I = I0 (2) I = 2I0 (3) I = 4I0 (4) I = I0/4

(1) 0.36 degrees (2) 0.18 degrees 17. A light has amplitude A and angle between analyser
and polariser is 60°. Light is transmitted by analyser
(3) 0.72 degrees (4) 0.09 degrees
has amplitude.
12. The main difference in the phenomenon of
interference and diffraction is that :- A
(1) A 2 (2)
2
(1) diffraction is due to interaction of light from the
3A A
same wavefront whereas interference is the (3) (4)
2 2 2
interaction of waves from two isolated sources.
18. Two Nicols are oriented with their principal planes
(2) diffraction is due to interaction of light from
making an angle of 60°. The percentage of incident
wavefront, whereas the interference is the
unpolarized light which passes through the system
interaction of two waves derived from the same
is :
source.
(1) 50% (2) 100%
(3) diffraction is due to interaction of waves derived
from the same source, whereas the interference (3) 12.5% (4) 37.5%
is the ben ding of light from the same 19. Two waves Y1 = asinwt and Y2 =asin(wt+d) are
wavelength. producing interference, then resultent intensity is –
(4) diffraction is caused by reflected waves from a (1) a2cos2 d/2 (2) 2a2cos2 d/2
source wheras interference caused is due to
(3) 3a2cos2 d/2 (4) 4a2cos2 d/2
refraction of waves from a surface.
20. In YDSE, the intensity of central bright fringe is
node06\B0AI-BO\Kota\JEE(MAIN)\Booster Course Sheet\Phy\Eng\Heat, Wave & Modern Physics\05-Wave Optics.p65

13. A ray of light strikes a glass plate at an angle of


8 mW/m2. What will be the intensity at l/6 path
60°. If the reflected and refracted rays are
difference?
perpendicular to each other, the index of refraction
of glass is :- (1) 8 mW/m2 (2) 6 mW/m2

(3) 4 mW/m2 (4) 2 mW/m2


(1) 3 (2) 3/2 (3) 3/2 (4) 1/2
21. Young's double slit experment is first performed in
14. Two coherent sources of equal intensities produce air and then in a medium other than air. It is found
a maximum of 100 units. If the amplitude of one of that 8th bright fringe in the medium lies where 5th
the sources is reduced by 20%, then the maximum dark fringe lies in air. The refractive index of the
intensity produced will be : medium is nearly :-
(1) 100 (2) 81 (3) 89 (4) 60 (1) 1.59 (2) 1.69 (3) 1.78 (4) 1.25

30 E
Wave Optics
ALLEN
22. In YDSE distance between slits and screen is 1.5m. 27. When the angle of incidence on a material is 60°,
When light of wavelength 500 nm is used then 2 nd the reflected light is completely polarized. The
bright fringe is obtained on screen at a distance of velocity of the refracted ray inside the material is
(in ms–1) :
10 mm from central bright fringe. What will be shift
in position of 2nd bright fringe if light of wavelength æ 3 ö
(1) 3 × 108 (2) çè ÷ × 108
550 nm is used. 2ø
(3) 3 × 10
8
(4) 0.5 × 108
(1) 2 mm (2) 1 mm
28. A light source of 5000Å wave length produces a
(3) 1.5 mm (4) 1.1 mm
single slit diffraction. The first minima in diffraction
23. In a double slit experiment, the two slits are pattern is seen, at a distance of 5mm from central
1 mm apart and the screen is placed 1 m away. maxima. The distance between screen and slit is
A monochromatic light of wavelength 500 nm is 2 metre. The width of slit in mm will be :
used. What will be the width of each slit for obtaining (1) 0.1 (2) 0.4 (3) 0.2 (4) 2
ten maxima of double slit within the central maxima 29. In Young's experiment, monochromatic light
of single slit pattern ? through a single slit S is used to illuminate the two
slits S1 and S2. Interference fringes are obtained on
(1) 0.1 mm (2) 0.5 mm a screen. The fringe width is found to be w. Now a
thin sheet of mica (thickness t and refractive index
(3) 0.02 mm (4) 0.2 mm
m) is placed near and in front of one of the two slits.
24. In a Young's double slit experiment with sodium Now the fringe width is found to be w¢, then :
light, slits are 0.589 m apart. The angular (1) w¢ = w/m (2) w¢ = wm
separation of the third maximum from the central (3) w¢ =(m – 1) tw (4) w¢ = w
maximum will be (given l = 589 nm) :– 30. In Young's experiment, light of wavelength 6000Å
(1) sin (0.33 × 10 )
–1 8
(2) sin (0.33 × 10 )
–1 –6 is used to produce fringes of width 0.8 mm at a
(3) sin–1 (3 × 10–8) (4) sin–1 (3 × 10–6) distance of 2.5 m. If the whole experiment is deep
25. A beam of light of l = 600 nm from a distant source in a liquid of refractive index 1.6, then fringe width
will be :
falls on a single slit 1 mm wide and the resulting
(1) 0.5 mm (2) 0.6 mm
diffraction pattern is observed on a screen 2 m
(3) 0.4 mm (4) 0.2 mm
away. The distance between first dark fringes on
either side of the central bright fringe is :-
(1) 1.2 cm (2) 1.2 mm
(3) 2.4 cm (4)2.4 mm
26. In Young's double slit experiment, the slits are 2mm
apart and are illuminated by photons of two
wavelengths l1 = 12000Å and l2 = 10000Å. At
node06\B0AI-BO\Kota\JEE(MAIN)\Booster Course Sheet\Phy\Eng\Heat, Wave & Modern Physics\05-Wave Optics.p65

what minimum distance from the common central


bright fringe on the screen 2m from the slit will a
bright fringe from one interference pattern coincide
with a bright fringe from the other ?
(1) 3 mm (2) 8 mm (3) 6 mm (4) 4 mm

ANSW ER KEY Exercise-I


Que. 1 2 3 4 5 6 7 8 9 10
Ans. 2 2 3 1 1 1 3 2 2 3
Que. 11 12 13 14 15 16 17 18 19 20
Ans. 1 2 1 2 4 3 4 3 4 2
Que. 21 22 23 24 25 26 27 28 29 30
Ans. 3 2 4 4 4 3 3 3 4 1
E 31
JEE(Main)-Physics
ALLEN
PREVIOUS YEARS' QUESTIONS EXERCISE-II
1. Electromagnetic waves are transverse in nature is 8. A mixture of light, consisting of wavelength 590
evident by- [AIEEE - 2002] nm and an unknown wavelength, illuminates Young's
(1) polarization (2) interference double slit and gives rise to two overlapping
interference patterns on the screen. The central
(3) reflection (4) diffraction
maximum of both lights coincide. Further, it is
2. To demonstrate the phenomenon of interference observed that the third bright fringe of known light
we require two sources which emit radiations of- coincides with the 4th bright fringe of the unknown
(1) nearly the same frequency [AIEEE - 2003] light. From this data, the wavelength of the unknown
light is :- [AIEEE - 2009]
(2) the same frequency
(1) 442.5 nm (2) 776.8 nm
(3) different wavelength
(3) 393.4 nm (4) 885.0 nm
(4) the same frequency and having a constant
9. At two points P and Q on screen in Young's double
phase relationship
slit experiment, waves from slits S1 and S2 have a
3. The maximum number of possible interference
maxima for slit-separation equal to twice the l
path difference of 0 and respectively. the ratio
wavelength in Young's double-slit experiment, is- 4

(1) infinite (2) five [AIEEE - 2004]


of intensities at P and Q will be : [AIEEE - 2011]

(3) three (4) zero (1) 3 : 2 (2) 2 : 1

4. A Yo ung's do ub le slit experiment us es a (3) 2:1 (4) 4 : 1


monochromatic source. The shape of the 10. In a Young's double slit experiment, the two slits act
interference fringes formed on a screen is- as coherent sources of waves of equal amplitude A
[AIEEE - 2005] and wavelength l. In another experiment with the
(1) hyperbola (2) circle same arrangement the two slits are made to act as
(3) straight line (4) parabola incoherent sources of waves of same amplitude and
wavelength. If the intensity at the middle point of
5. When an unpolarized light of intensity I0 is incident
the screen in the first case is I1 and in the second
on a polarizing sheet, the intensity of the light which
I1
does not get transmitted is- [AIEEE - 2005] case I2, then the ratio is :-
I2
1 1 [AIEEE - 2011]
(1) I (2) I (3) zero (4) I0
2 0 4 0 (1) 4 (2) 2 (3) 1 (4) 0.5
6. If I0 is the intensity of the principle maximum in 11. In Young's double slit interference experiment, the
the single slit diffraction pattern, then what will be slit widths are in the ratio 1 : 25. Then the ratio of
its intensity when the slit width is doubled- intensity at the maxima and minima in the
[AIEEE - 2005]
interference pattern is :- [AIEEE - 2012 (Online)]
node06\B0AI-BO\Kota\JEE(MAIN)\Booster Course Sheet\Phy\Eng\Heat, Wave & Modern Physics\05-Wave Optics.p65

(1) 1 : 5 (2) 3 : 2 (3) 9 : 4 (4) 1 : 25


I0 12. In a Young's double slit experiment with light of
(1) 2I0 (2) 4I0 (3) I0 (4)
2 wavelength l, fringe pattern on the screen has fringe
7. In a Young's double slit experiment the intensity at width b. When two thin transparent glass (refractive
inde x m) plates of thickness t 1 and t 2
l (t1 > t2) are placed in the path of the two beams
a point where the path difference is (l being
6 respectively, the fringe pattern will shift by a
the wavelength of the light used) is I. If I0 denotes distance: [AIEEE - 2012 (Online)]

the maximum intensity, I/I0 is equal to-


b(m - 1) b(m - 1) æ t 1 ö
[AIEEE - 2007] (1) (t1 - t 2 ) (2) çè t ÷ø
l l 2

1 3 1 3
(1) (2) (3) (4) mb t 1 l
2 2 2 4 (3) l t (4) (m – 1) . (t1 + t2)
2 b
32 E
Wave Optics
ALLEN
13. The maximum number of possible interference 18. The magnetic field in a travelling electromagnetic
maxima for slit separation equal to 1.8 l, where l wave has a peak value of 20nT. The peak value of
is the wavelength of light, in a Young's double slit electric field strength is : [JEE(Main)-2013]

experiment is :- [AIEEE - 2012 (Online)] (1) 3 V/m (2) 6 V/m


(1) Infinite (2) Zero (3) 5 (4) 3 (3) 9 V/m (4) 12 V/m
14. The first diffraction minimum due to a single slit 19. Two beams, A and B, of plane polarized light with
diffraction is seen at q = 30° for a light of wavelength mutually perpendicular planes of polarization are
5000 A° falling perpendicularly on the slit. The width seen through a polaroid. From the position when
of the slit is :- [AIEEE - 2012 (Online)] the beam A has maximum intensity (and beam B
(1) 5 × 10–5 cm (2) 2.5 × 10–5 cm has zero intensity), a rotation of polaroid through
(3) 1.25 × 10–5 cm (4) 10 × 10–5 cm 30° makes the two beams appear equally bright. If
the initial intensitites of the two beams are IA and IB
15. Young's double slit experiment is carried out by
using green, red and blue light, one color at a time. IA
respectively, then equals : [JEE(Main)-2014]
IB
The fringe widths recorded are bG , b R and b B ,
1 3
respectively. Then [IIT - 2012] (1) 1 (2) (3) 3 (4)
3 2
(1) bG > b B > b R (2) b B > bG > b R 20. In the Young's double slit experiment using a
monochromatic light of wavelength l, the path
(3) b R > b B > bG (4) b R > bG > b B
difference (in terms of an integer n) corresponding
16. A beam of unpolarised light of intensity I0 is passed to any point having half the peak intensity is :-
through a polaroid A and then through another
[JEE Advanced 2013]
polaroid B which is oriented so that its principal
l l
plane makes an angle of 45° relative to that of A. (1) ( 2n + 1) (2) ( 2n + 1)
2 4
The intensity of the emergent light is :-
l l
[JEE(Main)-2013]
(4) ( 2n + 1) (4) ( 2n + 1)
8 16
(1) I0 (2) I0/2 (3) I0/4 (4) I0/8
21. In a Young's double slit experiment, slits are
17. Two coherent point sources S1 and S2 are separated
separated by 0.5 mm, and the screen is placed
by a small distance 'd' as shown. The fringes obtained
150 cm away. A beam of light consisting of two
on the screen will be : [JEE(Main)-2013]
wavelengths, 650 nm and 520 nm, is used to obtain
interference fringes on the screen. The least
d
node06\B0AI-BO\Kota\JEE(MAIN)\Booster Course Sheet\Phy\Eng\Heat, Wave & Modern Physics\05-Wave Optics.p65

distance from the common central maximum to the


S1 S 2 point where the bright fringes due to both the
D Screen
wavelengths coincide is : [JEE(Main)-2017]

(1) points (2) straight lines (1) 9.75 mm (2) 15.6 mm


(3) semi-circles (4) concentric circles (3) 1.56 mm (4) 7.8 mm

PREVIOUS YEARS QUESTIONS ANSWER KEY Exercise-II


Que. 1 2 3 4 5 6 7 8 9 10
Ans. 1 4 2 1 1 3 4 1 2 2
Que. 11 12 13 14 15 16 17 18 19 20
Ans. 3 1 4 4 4 3 4 2 2 2
Que. 21
Ans. 4

E 33
JEE ( Main) - Physics
ALLEN
PHOTO ELECTRIC EFFECT & MATTER WAVE EXERCISE-I
1. A 200 W sodium street lamp emits yellow light of
5. A beam of white light is incident normally on a plane
wavelength 0.6 µm. Assuming it to be 50% efficient
surface absorbing 70% of the light and reflecting
in converting electrical energy to light, the number
the rest. If the incident beam carries 10W of power,
of photon of yellow light it emits per second is :-
(1) 62 × 1020 (2) 3 × 1020 find the force exerted by it on the surface.
(3) 1.5 × 10 20
(4) 6 × 1018 (1) 4.33 × 10–8 N (2) 5.76 × 10–8 N
2. Across a photoelectric cell a source of variable (3) 10.11 × 10–6 N (4) None
potential difference is connected and the 6. When a photosensitive surface is illuminated with
photoelectric current is plotted against the applied light of wavelength l, the stopping potential is V.
potential difference. The graph in broken lines When the same surface is illuminated by light of
represents a curve of current vs applied potential
difference. If the frequency is increased and the V
wavelength 2l, stopping potential is . If the
intensity is reduced, the curve which may now 3
represents the situation is: threshold wavelength for the surface is n l/2, then
I n=
A (1) 2 (2) 4
B
(3) 8 (4) 12
C
D 7. A metal plate is exposed to light with wavelength
V l. It is observed that electrons are ejected from the
(0,0)
surface of the plate. When a retarding uniform
(1) A (2) B (3) C (4) D electric field E is imposed, no electron can move
3. A silver ball of radius 4.8 cm is suspended by a thread away from the plate farther than a certain distance
in the vacuum chamber. UV light of wavelength
d. Then the threshold wavelength l0 for the material
200 nm is incident on the ball for some times during
of plate is (e is the electronic charge, h is Planck's
which a total energy of 1 ×10–7J falls on the surface.
constant and c is the speed of light) :-
Assuming on an average one out of 103 photons
incident is able to eject electron. The potential on -1 -1
æ 1 hc ö æ 1 eEd ö
sphere will be (1) l 0 = ç - ÷ (2) l 0 = ç - ÷
(1) 1 V (2) 2 V (3) 3 V (4) Zero è l eEd ø è l hc ø
4. In a photoelectric emission, electrons are ejected
from metals X and Y by light of frequency f. The hc eEd
(3) l 0 = l - (4) l 0 = l -
potential difference V required to stop the electrons eEd hc
is measured for various frequencies. If Y has a 8. When photons of energy hv fall on an aluminium
node06\B0AI-BO\Kota\JEE(MAIN)\Booster Course Sheet\Phy\Eng\Heat, Wave & Modern Physics\06-Photo Electric Effect & Matter Wave.p65
greater work function than X, which graph
plate (of work function E0), photoelectrons of
illustrates the expected results :-
maximum kinetic energy K are ejected. If the
V V frequency of the radiation is doubled, the maximum
X X
kinetic energy of the ejected photoelectrons will be-
Y Y (1) 2K (2) K
(1) (2)
(3) K + hv (4) K + E0
9. When a certain metallic surface is illuminated with
0 0
0 f 0 f mono-chromatic light of wavelength l, the stopping
potential for photo electric current is 6 V0. When
V V the same surface is illuminated with light of
Y Y
wavelength 2l, the stopping potential is 2V0. The
X X
threshold wavelength of this surface for photoelectric
(3) (4)
effect is-
0 0 (1) 6 l (2) 4l/3
0 f 0 f (3) 4l (4) 8l

34 E
Photo electric Effect & Matter Wave
ALLEN
10. A source of light is placed at a distance of 20 cm 14. In an experiment on photoelectric effect, the emitter
from a photo cell and stopping potential is found and the collector plates are placed at a separation
d and are connected through an ammeter without
to be 0.6 V. If the distance of source is now changed
any cell. A magnetic field B exists parallel to the
to 40 cm the stopping potential will be- plates (as shown). The work function of the emitter
(1) 0.3 V (2) 0.6 V is f and light incident on it has wavelength l. Then
the minimum value of B for which ammeter
(3) 1.2 V (4) 2.4 V registers zero current will be
11. Ultraviolet light of wavelength 300 nm and intensity (Take mass of electron as m and charge on it is e)
1.0 watt/m2 falls on the surface of a photosensitive
material. If 1 % of the incident photons produce
photoelectrons, then the number of photoelectrons ×× ×× ×××
d ×× ×× ××× A
emitted from an area of 1.0 cm2 of the surface- S ×× ×× ×××
(1) 9.61 × 1014 per sec

(2) 4.12 × 1013 per sec

(3) 1.51 × 1012 per sec 1 æ hc ö 2 æ hc ö


(1) m ç - f÷ (2) 2m ç - f ÷
(4) 2.13 × 1011 per sec
2d è l ø ed è l ø
12. A sensor is exposed for time t to a lamp of power
P placed at a distance l. The sensor has an opening
1 æ hc f ö 2 æ 2hc ö
(3) mç - ÷ (4) mç - f÷
that is 4d in diameter. Assuming all energy of the
ed è l 2ø ed è l ø
lamp is given off as light, the number of photons 15. In P.E.E, Kmax of electrons is 2.0 eV. If frequency
entering the sensor if the wavlength of light is l is- of light is decreased by 40% then Kmax is 1.0 eV.
Work function of electron emitter is :-
(1) 4 eV (2) 0.5 eV (3) 2 eV (4) 1 eV
Pld 2 t 4Pld 2 t 16. A cathode emits 1.8 × 1014 electrons per second,
(1) N= (2) N=
hcl2 hcl2 when heated, when 400V is applied to anode all
the emitted electrons reach the anode. The charge
on electron is 1.6 × 10–19C. The maximum anode
Pld 2 t Pld 2 t current is :-
(3) N = (4) N = (1) 27mA (2) 29mA
4hcl2 16hcl1
(3) 72mA (4) 29mA
17. A totally reflecting small plane mirror placed
node06\B0AI-BO\Kota\JEE(MAIN)\Booster Course Sheet\Phy\Eng\Heat, Wave & Modern Physics\06-Photo Electric Effect & Matter Wave.p65

13. The intensity of sunlight on the surface of earth is


horizontally faces a parallel beam of light as shown
1400 W/m2. Assuming the mean wavelength of
in figure. The mass of mirror is 20 gm. Assume that
sunlight to be 6000 Å, calculate:– there is no absorption in the lens and that 30% of
(a) The photon flux arriving at 1 m2 area on earth the light emitted by the source goes through the lens.
Find the power of the source needed to support the
perpendicular to light radiations and
weight of the mirror (take g = 10 m/s2) :-
(b) The number of photons emitted from the sun
per second (Assuming the average radius of
Earth's orbit to be 1.49 × 1011 m)

(1) 4.22 × 1021 photons/sec, 1.18 × 1045

(2) 1.18 × 1045 photons/sec, 4.22 × 1021


S
(3) 2.22 × 1021 photons/sec, 1.18 × 1045

(4) 4.22 × 1021 photons/sec, 0.18 × 1045 (1) 80 MW (2) 100 MW


(3) 20 MW (4) 25 MW
E 35
JEE ( Main) - Physics
ALLEN
18. Electromagnetic wave of intensity 1400 W/m2 falls 24. With what potential an electron should be
on metal surface on area 1.5 m2 is completely accelerated so that its de Broglie wave-length
absorbed by it. Find out force exerted by beam. becomes equal to the wave length of second line of
(1) 14 × 10–5 N (2) 14 × 10–6 N Balmer series for He+ ion ?
(3) 7 × 10 N–5
(4) 7 × 10–6 N
19. A proton is accelerated through a potential 9R 2 h 2 162R 2 h 2 81R 2 h 2 32R 2 h 2
(1) (2) (3) (4)
difference of 400 V. To have same de-Broglie 32me 25me 32me 81me
wavelength, what potential difference must be 25. A particle of mass 'm' is projected from ground with
applied across doubly ionised 8O16 atom :-
velocity 'u' making angle q with the vertical. The de-
(1) 50 volt (2) 12.5 volt
Broglie wavelength of the particle at the highest
(3) 100 volt (4) 25 volt point is -
20. The de-Broglie wavelength of a particle moving with (1) ¥ (2) h/musinq
a velocity 2.25 ×108 m/s is equal to the wavelength
(3) h/mucosq (4) h/mu
of photon. The ratio of kinetic energy of the particle
26. The de–Broglie wavelength of a neutron at 27°C
to the energy of the photon is
is l. What will be its wavelength at 927°C?
(velocity of light is 3 × 108 m/s)
(1) 1/8 (2) 3/8 l l l l
(1) (2) (3) (4)
(3) 5/8 (4) 7/8 2 3 4 9
21. The ratio of de-Broglie wavelength of a-particle to 27. In Davisson-Germer experiment an electron beam
that of a proton being subjected to the same
of energy 60 eV falls normally on the surface of
magnetic field so that the radii of their path are
a crystal. If the maximum intensity is obtained at
equal to each other assuming the field induction
r an angle of 60° to the direction of incident beam,
vector B is perpendicular to the velocity vectors of then the inter-atomic distance in the lattice plane
the a-particle and the proton is of the crystal will be-
1 1 (1) 18 Å (2) 3.6 Å
(1) 1 (2) (3) (4) 2 (3) 1.8 Å (4) 0.18 Å
4 2
28. An a–particle moves in circular path of radius
22. An e-m wave of wavelength l is incident on a photo
0.83 cm in the presence of a magnetic field of
sensitive surface of negligible work function. If the
photoelectrons emitted from this surface have the 0.25 Wb/m2. Find the De Broglie wavelength
de-Broglie wavelength l1. Find relation between 'l' associated with the particle.
and 'l1' :- (1) 0.01 Å (2) 0.02 Å (3) 0.03 Å (4) 0.04 Å
29. Proton, deutron and a particles are accelerated
æ 2mC ö 2 æ mC ö 2 through the same potential difference. Then the
(1) l = ç ÷ l1 (2) l = ç ÷ l1
è h ø è 2h ø ratio of their de-Broglie wavelength as
node06\B0AI-BO\Kota\JEE(MAIN)\Booster Course Sheet\Phy\Eng\Heat, Wave & Modern Physics\06-Photo Electric Effect & Matter Wave.p65
æ 2mC ö 2 (1) 1: 2 :1 (2) 1 : 1 : 1
(3) l1 = ç ÷l (4) None
è h ø (3) 1: 2 : 2 2 (4) 2 2 : 2 :1
23. A particle of mass 3m at rest decays into two particle 30. An alpha particle and a proton are fired through
of masses m and 2m having non-zero velocities. The the same magnetic field which is perpendicular to
ratio of the de-Broglie wavelength of the particles their velocity vectors. Both move such that radius
(l1/l2) is :- of curvature of their paths is same. The ratio of their
(1) 1/2 (2) 1/4
de-Broglie wave lengths is :-
(3) 2 (4) None of these
(1) 2 : 3 (2) 3 : 4 (3) 5 : 7 (4) 1 : 2
ANSWER KEY Exercise-I
Que. 1 2 3 4 5 6 7 8 9 10
Ans. 2 4 3 1 1 3 2 3 3 2
Que. 11 12 13 14 15 16 17 18 19 20
Ans. 3 1 1 2 2 2 2 4 2 2
Que. 21 22 23 24 25 26 27 28 29 30
Ans. 3 1 4 1 2 1 3 1 4 4

36 E
Photo electric Effect & Matter Wave
ALLEN
PREVIOUS YEARS' QUESTIONS EXERCISE-II
PHOTO ELECTRIC EFFECT 4. Statement-1 : When ultraviolet light is incident on
1. The threshold frequency for a metallic surface a photocell, its stopping potential is V0 and the
corresponds to an energy of 6.2 eV and the maximum kinetic energy of the photoelectrons is
stopping potential for a radiation incident on this Kmax. When the ultraviolet light is replaced by
surface is 5V. The incident radiation lies in- X-rays, both V0 and Kmax increase.
[AIEEE - 2006]
Statement-2 : Photoelectrons are emitted with
(1) ultra-violet region
speeds ranging from zero to a maximum value
(2) infra-red region
because of the range of frequencies present in the
(3) visible region
incident light. [AIEEE - 2010]
(4) X-ray region
2. The surface of a metal is illuminated with the light (1) Statement–1 is true, Statement–2 is false.
of 400 nm. The kinetic energy of the ejected (2) Statement–1 is true, Statement–2 is true;
photoelectrons was found to be 1.68 eV. The work Statement–2 is the correct explanation of
function of the metal is : (hc = 1240 eV-nm) Statement–1
[AIEEE - 2009]
(3) Statement–1 is true, Statement–2 is true;
(1) 1.51 eV (2) 1.68 eV Statement–2 is not the correct explanation of
(3) 3.09 eV (4) 1.41 eV
Statement–1
3. Photoelectric effect experiments are performed
using three different metal plates p, q and r having (4) Statement–1 is false, Statement–2 is true
work functions fp = 2.0 eV, fq = 2.5 eV and 5. If a source of power 4kW produces
fr = 3.0 eV, respectively. A light beam containing 1020 photons/second, the radiation belongs to apart
wavelengths of 550 nm, 450 nm and 350 nm with of the spectrum called :- [AIEEE - 2010]
equal intensities illuminates each of the plates. The (1) g-rays
correct I-V graph for the experiment is :
(2) X-rays
[JEE-2009]
(3) ultraviolet rays
I
p (4) microwaves
q 6. This question has Statement 1, Statement 2. Of the
(1) four choices given after the statement, choose the
r
one that best describes the two statements.
V
Statement-1 : A metallic surface is irradiated by
I a monochromatic light of frequency v > v0 (the
threshold frequency). If the incident frequency is now
node06\B0AI-BO\Kota\JEE(MAIN)\Booster Course Sheet\Phy\Eng\Heat, Wave & Modern Physics\06-Photo Electric Effect & Matter Wave.p65

doubled, the photocurrent and the maximum kinetic


(2) energy are also doubled.
p
q Statement-2 : The maximum kinetic energy of
r
V photoelectrons emitted from a surface is linearly
dependent on the frequency of the incident light.
I
The photocurrent depends only on the intensity of
r
q the incident light. [AIEEE-2012 (Online)]
(3) p (1) Statement-1 is true, Statement-2 is true and
V Statement-2 is the correct explanation of
Statement-1.
I
(2) Statement-1 is true, Statement-2 is true and
Statement-2 is not the correct explanation
(4) of statement-1.
r q
p (3) Statement-1 is true, Statement-2 is false.
V
(4) Statement-1 is false, Statement-2 is true.
E 37
JEE ( Main) - Physics
ALLEN
7. Photons of an electromagnetic radiation has an 10. The work functions of Silver and sodium are 4.6
energy 11 keV each. To which region of and 2.3 eV, repetitively. The ratio of the slope of
electromagnetic spectrum does it belong ? the stopping potential versus frequency plot for
Silver to that of Sodium is. [JEE Advanced 2013]
[JEE(Main)-2013 (Online)]
(1) 1 (2) 2
(1) Ultra violet region
(3) 3 (4) 4
(2) Infrared region

(3) visible region 11. The radiation corresponding to 3 ® 2 transition of


hydrogen atom falls on a metal surface to produce
(4) X- ray region
photoelectrons. These electrons are made to enter
8. A copper ball of radius 1 cm and work function a magnetic field of 3 × 10–4T. If the radius of the
4.47eV is irradiated with ultraviolet radiation of largest circular path followed by these electrons is
wavelength 2500 Å. The effect of irradiation results 10.0 mm, the work function of the metal is close
in the emission of electrons from the ball. Further to :- [JEE(Main) - 2014]
the ball will acquire charge and due to this there (1) 0.8 eV (2) 1.6 eV
will be a finite value of the potential on the ball. The
charge acquired by the ball is : (3) 1.8 eV (4) 1.1 eV

[JEE(Main)-2013 (Online)]
12. A metal surface is illuminated by light of two
different wavelength 248 nm and 310 nm. The
(1) 2.5× l0–11 C (2) 4.5 × 10–12 C maximum speeds of t he phot oelectro ns
(3) 7.5× 10–13C (4) 5.5 × l0–13 C corresponding to these wavelengths are u1 and u2,
respectively. If the ratio u 1 : u2 = 2 : 1 and
9. In an experiment on photoelectric effect a student
hc = 1240 eV nm, the work function of the metal
plots stopping potential VO against reciprocal of the
is nearly [JEE Advanced 2014]
wavelength l of the incident light for two different
metals A and B. These are shown in the figure. (1) 3.7 eV (2) 3.2 eV

(3) 2.8 eV (4) 2.5 eV

13. Match List-I (Fundament Experiment) with List-II (its


VO metal A metal B conclusion) and select the correct option from the
choices given below the list : [JEE Main-2015]

1 List-I List-II node06\B0AI-BO\Kota\JEE(MAIN)\Booster Course Sheet\Phy\Eng\Heat, Wave & Modern Physics\06-Photo Electric Effect & Matter Wave.p65

l
(A) Franck-Hertz (i) Particle
Looking at the graphs, you can most appropriately Experiment. nature of light
say that : [JEE(Main)-2013 (Online)]
(B) Photo-electric (iii) Discrete
(1) Work function of metal B is greater than that experiment energy levels
of metal A of atom

(2) Students data is not correct (C) Davison-Germer (iiii) Wave nature
Experiment of electroc
(3) For light certain wavelength falling on both
metals, maximum kinetic energy of electrons (iv) Structure of
atom
emitted from A will be greater than those
emitted from B

(4) Work function of metal A is greater than that (1) A-ii, B-i, C-iii (2) A-iv, B-iii, C-ii
of metal B (3) A-i, B-iv, C-iii (4) A-ii, B-iv, C-iii

38 E
Photo electric Effect & Matter Wave
ALLEN
14. Radiation of wavelength l, is incident on a photocell. MATTER WAVE
The fastest emitted electron has speed v. If the 16. A proton has kinetic energy E = 100 keV which is
3l equal to that of a photon. The wavelength of photon
wavelength of changed to , the speed of the is l2 and that of proton is l1. The ratio of l1 / l2
4
is proportional to [JEE-2004 (Scr.)]
fastest emitted electron will be :- [JEE(Main) - 2016]
(1) E2 (2) E1/2
1/ 2
æ3ö (3) E–1 (4) E –1/2
(1) = v ç ÷
è5ø
17. Electrons with de-Broglie wavelength l fall on the
1/ 2
æ4ö target in an X-ray tube. The cut-off wavelength of
(2) > v ç ÷ the emitted X-rays is
è3ø [JEE 2007]

1/ 2
æ4ö 2mcl2 2h
(3) < v ç ÷ (1) l0 = (2) l0 =
è3ø h mc
1/ 2
æ4ö
(4) = v ç ÷
è3ø 2m 2c 2l3
(3) l0 = (4) l0 = l
15. An electron beam is accelerated by a potential h2
difference V to hit a metallic target to produce
X-rays. It produces continuous as well as Directions : Questions No. 18, 19 and 20 are based
characteristic X-rays.If lmin is the smallest possible on the following paragraph.
wavelength of X-ray in the spectrum, the variation Wave property of electrons implies that they will
of log lmin with log V is correctly represented in : show diffraction effects. Davission and Germer
[JEE Main-2017]
demonstrated this by diffracting electrons from
crystals. The law governing the diffraction from a
crystal is obtained by requiring that electron waves
log lmin reflected from the planes of atoms in a crystal
(1) interfere constructively (see figure)
log V
Incoming Outgoing
Electrons i Electrons
node06\B0AI-BO\Kota\JEE(MAIN)\Booster Course Sheet\Phy\Eng\Heat, Wave & Modern Physics\06-Photo Electric Effect & Matter Wave.p65

log lmin d

(2)
Crystal plane
log V
18. If a strong diffraction peak is observed when
electrons are incident at an angle 'i' from the normal
log lmin to the crystal planes with distance 'd' between them
(3) (see figure), de-Broglie wavelength ldB of electrons
log V can be calculated by the relationship (n is an
integer): [AIEEE - 2008]

(1) d sin i = n ldB


log lmin
(2) 2d cos i = n ldB
(4)
log V (3) 2d sin i = n ldB

(4) d cos i = n ldB


E 39
JEE ( Main) - Physics
ALLEN
19. Electrons accelerated by potential V are 22. After absorbing a slowly moving neutron of mass mN
diffracted from a crystal. If d = 1Å and i = 30° (mo ment um ~ 0) a nucleus of mass M
V should be about (h = 6.6 × 10 –34 Js , breaks into t wo nu clei of masses m 1 and
me = 9.1 × 10–31 kg, e = 1.6 × 10–19 C) : 5m1(6m1 = M + mN), respectively. If the de Broglie
[AIEEE - 2008] wavelength of the nucleus with mass m1 is l, then
de Broglie wavelength of the other nucleus will
(1) 2000 V (2) 50 V
be :- [AIEEE - 2011]
(3) 500 V (4) 1000 V
(1) 25 l (2) 5l
20. In an experiment, electrons are made to pass
l
through a narrow slit of width 'd' comparable to (3) (4) l
their de-Broglie wavelength. They are detected on
5
a screen at a distance 'D' from the slit (see figure). 23. This question has Statement-1 and Statement-2. Of
the four choices given after the statements, choose
the one that best describes the two statements.

Statement-1: Davisson-Germer experiment


d y=0 established the wave nature of electrons.

Statement-2 : If electrons have wave nature, they


D
can interfere and show diffraction. [AIEEE - 2012]

(1) Statement-1 is true, Statement-2 is true and


Which of the following graphs can be expected
Statement-2 is not the correct explanation of
to represent the number of electrons 'N' detected
Statement-1.
as a function of the detector position 'y'
(2) Statement-1 is false, Statement-2 is true
(y = 0 corresponds to the middle of the slit) ?
[AIEEE - 2008] (3) Statement-1 is true, Statement-2 is false

(4) Statement-1 is true, Statement-2 is true and


y y
Statement-2 is the correct explanation of
statement-1.
(1) N d (2) N d

24. The frequency of X-rays, g-rays and ultraviolet rays


are respectively a, b and c then :-

[AIEEE-2012 (Online)]
node06\B0AI-BO\Kota\JEE(MAIN)\Booster Course Sheet\Phy\Eng\Heat, Wave & Modern Physics\06-Photo Electric Effect & Matter Wave.p65
y y
(1) a < b ; b > c
(3) N d (4) N d
(2) a > b ; b > c

(3) a < b < c

21. An a–particle and a proton are accelerated from (4) a = b = c


rest by a potential difference of 100 V. After this, 25. If the kinetic energy of an electron is increased four
their de Broglie wavelengths are l a and l p times, the wavelength of the de-Broglie wave
associated with it would become :-
lp
respectively. The ratio , to the nearest integer,, [AIEEE-2012 (Online)]
la
(1) Two times
is [JEE 2010]
(2) Half
(1) 1 (2) 2
(3) One fourth
(3) 3 (4) 4 (4) Four times
40 E
Photo electric Effect & Matter Wave
ALLEN
26. If the radius of first orbit of H atom is a0, the de- 29. Electrons are accelerated through a potential
Broglie wavelength of an electron in the third orbit difference V and protons are accelerated through a
is :- [AIEEE-2012 (Online)] potential difference 4V. The de-Broglie wavelengths
(1) 6 p a0 (2) 8 p a0 are le and lp for electrons and protons respectively.

(3) 2 p a0 (4) 4 p a0 le
The ratio of is given by :
27. A proton is fired from very far away towards a lp
nucleus with charge Q = 120 e, where e is the
electronic charge. It makes a closest approach of (Given me is mass of electron and mp is mass of
10 fm to the nucleus. The de Broglie wavelength proton). [JEE(Main)-2013 (Online)]

(in units of fm) of the proton at its start is


l me le mp
(1) l =
e
(Take : The proton mass, mP = (5/3) × 10–27 kg; (2) =2
p mp lp me

–15
1 9
h/e = 4.2 × 10 J.s/C;
4pe 0 = 9 × 10 m/F; le mp l e 1 me
(3) = (4) =
1 fm = 10 –15
m) [JEE 2012] lp me l p 2 mp

(1) 7 (2) 8 30. A particle A of mass m and initial velocity v collides


(3) 9 (4) 10
m
with a particle B of mass which is at rest. The
28. Orbits of a particle moving in a circle are such that 2
the perimeter of the orbit equals an integer number
collision is head on, and elastic. The ratio of the
of de-Broglie wavelengths of the particle. For a
de–Broglie wavelengths lA to lB after the collision
charged particle moving in a plane perpendicular
is: [JEE Main-2017]
to a magnetic field, the radius of the n th orbital will
therefore be proportional to : lA 2 lA 1
(1) = (2) =
[JEE(Main)-2013 (Online)] lB 3 lB 2
(1) n2 (2) n1/2
lA 1 lA
(3) n1/4 (4) n (3) = (4) =2
lB 3 lB
node06\B0AI-BO\Kota\JEE(MAIN)\Booster Course Sheet\Phy\Eng\Heat, Wave & Modern Physics\06-Photo Electric Effect & Matter Wave.p65

PREVIOUS YEARS QUESTIONS ANSWER KEY Exercise-II


Que. 1 2 3 4 5 6 7 8 9 10
Ans. 1 4 1 1 2 4 4 4 1,2,3 1
Que. 11 12 13 14 15 16 17 18 19 20
Ans. 4 1 1 2 3 2 1 2 2 4
Que. 21 22 23 24 25 26 27 28 29 30
Ans. 3 4 4 1 2 1 1 2 2 4
E 41
JEE ( Main) - Physics
ALLEN
NUCLEAR PHYSICS & RADIOACTIVITY EXERCISE-I
1. A radioactive reaction is 92U238 ® 82Pb206. How 7. A typical nuclear fusion reaction is given by :-
many a and b particles are emitted.
1H
2 + 1H 2 ® 1H3 + 1H1 + 4.0 MeV
(1) 10a, 6b (2) 4 protons, 8 neutrons
1H
2 + 1H2 ® 2He4 + 0n1 + 17.6 MeV
(3) 6 electrons 8 protons (4) 6b, 8a
The net result of two reactions is :-
2. In the given nuclear reaction A, B, C, D, E
3 × 1H2 ® 2He4 + 0n1 + 1H1 + 21.6 MeV
represents
The energy released per nucleon of the reactant,
238
a A b C E 234 in the above thermonuclear reaction is :-
92 U B Th DPa 92 U
(1) 1.8 MeV (2) 3.6 MeV
(1) A = 234, B = 90, C = 234, D = 91, E = b
(3) 7.2 MeV (4) 21.6 MeV
(2) A = 234, B = 90, C = 238, D = 94, E = a

(3) A = 238, B = 93, C = 234, D = 91, E = b 8. The rest mass of the deuteron, 12 H, is equivalent to

(4) A = 234, B = 90, C = 234, D = 93, E = a an energy of 1876 MeV, the rest mass of a proton
is equivalent to 939 MeV and that of a neutron to
3. The stable nucleus which has a radius half of Fe56 940 MeV. A deuteron may disintegrate to a proton
is - and a neutron if it :
(1) Ca40 (2) S16 (1) emits a g-ray photon of energy 2 MeV
(3) Na 21
(4) Li7
(2) captures a g-ray photon of energy 2 MeV
4. The binding energies of the atoms of elements A (3) emits a g-ray photon of energy 3 MeV
and B are Ea and Eb respectively. Three atoms of
(4) captures a g-ray photon of energy 3 MeV
the element B fuse to give one atom of element A.
This fusion process is accompained by release of 9. If there is a mass defect of 0.1% in nuclear fission,
energy e. Then, Ea, Eb and e are related to each then the energy released in the fission of 1 kg mass
other as : would be-

(1) Ea + e = 3Eb (2) Ea = 3Eb (1) 2.5 × 105 kWh (2) 2.5 × 107 kWh

(3) Ea – e = 3Eb (4) Ea + 3Eb + e = 0 (4) 2.5 × 109 kWh (4) 2.4 × 10–7 kWH

5. If 200 MeV energy is released in the fission of a 10. If the average number of neutrons liberated per node06\B0AI-BO\Kota\JEE(MAIN)\Booster Course Sheet\Phy\Eng\Heat, Wave & Modern Physics\07-Nuclear Physics & Radioactivity.p65

single U235 nucleus, the number of fissions required fission is 2.5 and energy released per fission is
per second in a nuclear reactor are 250 MeV then the number of neutrons generated
3.125 × 1013, find the power of the reactor. per second in a nuclear reactor of 100 MW, will be :-

(Given 1 eV = 1.6 ´10-19 J ) (1) 2.5 × 1018 (2) 2.5 × 1019

(3) 6.25 × 1018 (4) 6.25 × 1019


(1) 1 Kilo watt (2) 2 Kilo watt
11. The half life of a radioactive substance is 20 minutes.
(3) 4 Kilo watt (4) 2.5 Kilo watt
The approximate time interval (t2 – t1) between the
6. A nuclear reactor delivers a power of 10 9 W. What 4
time t2 when of it has decayed and time t1 when
is the amount of fuel consumed by the reactor in 5
one hour ?
1
of it had decayed is :-
(1) 0.04 g (2) 0.08 g 5
(3) 0.72 g (4) 0.96 g
(1) 20 min (2) 28 min (3) 40 min (4) 14 min
42 E
Nuclear Physics & Radioactivity
ALLEN
12. Half life of a radioactive substance A is twice the 17. A radioactive element emits 200 particles per
half life of substance B. Initially the number of nuclei second. After three hours 25 particles per second
of A and B are NA and NB respectively. After three are emitted. The half life period of element will be
half life of A, activities of both the samples are
(1) 50 minutes (2) 60 minutes
N
equal. The ratio A is :- (3) 70 minutes (4) 80 minutes
NB
18. The half life of the isotope 11Na24 is 15 hrs. How

1 1 1 1 7
(1) (2) (3) (4) much time does it take for th fo a sample of this
4 8 3 6 8
13. Decay rate for a certain mass of a radioactive isotope to decay :-
substance measured at different times varies with
(1) 75 hrs (2) 65 hrs
time as shown in fig. The number of active nuclei at
t = 8 hr will be :- (3) 55 hrs (4) 45 hrs
(1) 12.5 200 19. 16 gm sample of a radioactive element is taken
Decay/sec

(2) 25 150 from Bombay to Delhi in 2 hour and it was found


100 that 1 gm of th e elemen t remained
(3) 9× 104 50 (undisintegrated). Half life of the element is
(4) 1.3 × 105 2 4 t(hr)
1 1
14. In a radioactive element the fraction of initial amount (1) 2 hour (2) 1 hour (3) hour (4) hour
2 4
remaining after its mean life time is
20. The activity of a sample of a radioactive material is
1 1 1 1 A, at time t1 and A2 at time t2 (t2 > t1). if its mean
(1) 1 – (2) 2 (3) (4) 1– 2
e e e e life T, then

15. Activity of a radioactive substance is R1 at time t1 (1) A1t1 = A2t2 (2) A1– A2 = t2t1

R2 (3) A 2 = A1e (t 1 – t 2 ) / T (4) A 2 = A1e(t 1 / t 2 ) / T


and R2 at time t2(t2 > t1). Then the ratio is:
R1
21. A nucleus with mass number 220 initially at rest
node06\B0AI-BO\Kota\JEE(MAIN)\Booster Course Sheet\Phy\Eng\Heat, Wave & Modern Physics\07-Nuclear Physics & Radioactivity.p65

emits an a - particle. If the Q value of the reaction


t2 - l ( t1 + t 2 )
(1) (2) e is 5.5 MeV, calculate the kinetic energy of the
t1
a - particle.

(1) 4.4 MeV (2) 5.4 MeV


æ t1 - t 2 ö l ( t1 - t 2 )
(3) e ç ÷ (4) e (3) 5.6 MeV (4) 6.5 MeV
è l ø
22. A and B are two radioactive substances whose half
16. The activity of a sample reduces from A 0 to
lives are 1 and 2 years respectively. Initially 10 gm of
A0 / 3 in one hour. The activity after 3 hours A and 1 gm of B is taken. The time (approximate)

more will be :- after which they will have same quantity remaining is

(1) 6.62 years (2) 5 years


A0 A0 A0 A0
(1) (2) (3) (4) (3) 3.2 years (4) 7 years
3 3 9 9 3 27
E 43
JEE ( Main) - Physics
ALLEN
23. The half life of a radioactive isotope 'X' is 50 years. 28. A gamma ray photon creates an electron-positron
It decays to another element 'Y' which is stable. pair. If the rest mass of electron is 0.5 MeV and the
The two elements 'X' and 'Y' were found to be in total KE of electron-positron pair is 0.78 MeV, the
the ratio of 1 : 15 in a sample of a given rock. energy of gamma ray photon must be :
The age of the rock was estimated to be :-
(1) 0.78 MeV (2) 1.78 MeV
(1) 150 years (2) 200 years
(3) 1.28 MeV (4) 0.28 MeV
(3) 250 years (4) 100 years
29. A radioactive sample consists of two distinct species
24. In an a-decay kinetic energy of a particle is having equal number of atoms N0 initially. The
98 MeV and Q-value of the reaction is 100 MeV. mean-life of one species is t and of the other is 5t.
The mass number of the mother nucleus is. (Assume The decay products in both cases is stable. The
that daughter nucleus is in ground state):- total number of radioactive nuclei at t = 5t is:-

(1) 100 (2) 200


æ e5 + 1 ö æ e4 + 1 ö
(3) 300 (4) None of these (1) N0 ç 5 ÷ (2) N0 ç 5 ÷
è e ø è e ø
25. The ratio of molecular mass of two radioactive

3
substances is and the ratio of their decay constants æ e + e5 ö
2 (3) N 0 ç (4) N0e–3
5 ÷
è e ø
4
is . Then the ratio of their initial activity per mole
3 30. Samples of two radioactive nuclides, X and Y, each
will be :- have equal activity A 0 at time t = 0. X has
half–life of 24 years and Y a half–life of 16 years.
(1) 2 (2) 8/9 (3) 4/3 (4) 9/8
The samples are mixed together. What will be the
26. A radioactive element A with a half-value period of total activity of the mixture at t = 48 years?
2 hours decays giving a stable element Y. After a
time the ratio of X to Y atoms is 1 : 7. Then, t is: 1 1
(1) A0 (2) A0
(1) 6 hour (2) 4 hour 2 4

(3) between 4 and 6 hour (4) 14 hour


3 3
27. Two radioactive materials X1 and X2 contain same (3) A0 (4) A0
16 8
number of nuclei. If 6ls–1 and 4ls–1 are the decay node06\B0AI-BO\Kota\JEE(MAIN)\Booster Course Sheet\Phy\Eng\Heat, Wave & Modern Physics\07-Nuclear Physics & Radioactivity.p65

constants of X1 and X2 respectively, the ratio of


number of nuclei, undecayed of X1 to that of X2 will
be (1/e) after a time :-

1 1 1 1
(1) sec (2) sec (3) sec (4) sec
2l 10l 5l l

ANSWER KEY Exercise-I


Que. 1 2 3 4 5 6 7 8 9 10
Ans. 4 1 4 3 1 1 2 4 2 3
Que. 11 12 13 14 15 16 17 18 19 20
Ans. 3 1 4 3 4 1 2 4 3 3
Que. 21 22 23 24 25 26 27 28 29 30
Ans. 2 1 2 2 3 1 1 2 2 4

44 E
Nuclear Physics & Radioactivity
ALLEN
PREVIOUS YEARS' QUESTIONS EXERCISE-II
NUCLEAR PHYSICS 8. If a star converts all of its Helium into oxygen
1. In the nuclear fusion reaction, nucleus, find the amount of energy released per
2
nucleus of oxygen. O = 15.9994 amu and
1
H + 13 H ® 24 He + n He = 4.0026 amu [JEE' 2005 (Scr)]

given that the repulsive potential energy between (1) 7.26 MeV (2) 7 MeV
the two nuclei is 7.7 × 10–14 J, the temperature at (3) 10.24 MeV (4) 5.12 MeV
which the gases must be heated to initiate the 1
reaction i s nearly [Bo ltzmann's constant 9. An alpha nucleus of energy mv2 bombards a
2
k = 1.38 × 10–23 J/K]- [AIEEE - 2003]
heavy nuclear target of charge Ze. Then the
7
(1) 10 K (2) 10 K 5 3
(3) 10 K (4) 109 K distance of closest approach for the alpha nucleus
2. A nucleus with Z = 92 emits the following in a will be proportional to- [AIEEE - 2006]
sequence : a, a, b–, b–, a, a, a, a, b–, b–, a, b+, (1) v2 (2) 1/m
b+, a. The Z of the resulting nucleus is- (3) 1/v4 (4) 1/Ze
[AIEEE - 2003] 10. When 3Li7 nuclei are bombarded by protons and
(1) 76 (2) 78 (3) 82 (4) 74 the resultant nuclei are 4Be8, the emitted particles
3. When U 238 nucleus originally at rest, decays by will be- [AIEEE - 2006]
emitting an alpha particle having a speed u, the (1) alpha particles (2) beta particles
recoil speed of the residual nucleus is- (3) gamma photons (4) neutrons
[AIEEE - 2003]
11. If the binding energy per nucleon in 73 Li and 24 He
4u 4u 4u 4u
(1) (2) – (3) (4) – nuclei are 5.60 MeV and 7.06 MeV respectively,
238 234 234 238
then in the reaction :
4. A nucleus disintegrates into two nuclear parts which
have their velocities in the ratio 2 : 1 The ratio of p + 73 Li ® 2 24 He
their nuclear sizes will be- [AIEEE - 2004]
energy of proton must be- [AIEEE - 2006]
(1) 21/3 : 1 (2) 1 : 31/2
(1) 28.24 MeV (2) 17.28 MeV
(3) 31/2 : 1 (4) 1 : 21/3
(3) 1.46 MeV (4) 39.2 MeV
5. The binding energy per nucleon of deuteron (12 H) 12. If M0 is the mass of an oxygen isotope 8O17,
Mp and Mn are the masses of a proton and a
and helium nucleus (24 He) is 1.1 MeV and 7 MeV
neutron, respectively, the nuclear binding energy
respectively. If two deuteron nuclei react to form a of the isotope is- [AIEEE - 2007]
single helium nucleus, then the energy released is- (1) (Mo – 8Mp)c 2
node06\B0AI-BO\Kota\JEE(MAIN)\Booster Course Sheet\Phy\Eng\Heat, Wave & Modern Physics\07-Nuclear Physics & Radioactivity.p65

[AIEEE - 2004] (2) (8Mp + 9Mn – Mo)c2


(1) 13.9 MeV (2) 26.9 MeV (3) Moc2
(3) 23.6 MeV (4) 19.2 MeV (4) (Mo – 17Mn)c2
6. If radius of the 27 13. In the options given below, let E denote the rest
Al nucleus is estimated to be
13 mass energy of a nucleus and n a neutron. The
125 correct option is :- [JEE-2007]
3.6 fermi, then the radius of Te nucleus be
( ) ( I) + E ( Y ) + 2E(n)
52
236 137 97
nearly- [AIEEE - 2005] (1) E 92 U >E 53 39
(1) 6 fermi (2) 8 fermi
(2) E ( U) < E ( I ) + E ( Y ) + 2E(n)
236 137 97
(3) 4 fermi (4) 5fermi
92 53 39
7. A nuclear transfo rmat io n is deno ted by

(3) E ( U ) < E ( Ba ) + E ( Kr ) + 2E(n)


7 236 140 94
X(n, a) ® 3 Li . Which of the following is the nucleus 92 56 36

(4) E ( U) = E ( Ba ) + E ( Kr ) + 2E(n)
of element X ? [AIEEE - 2005]
236 140 94
12 10 9 11 92 56 36
(1) 6
C (2) 5
B (3) 5
B (4) 4
Be
E 45
JEE ( Main) - Physics
ALLEN
14. This question contains Statement-1 and Statement-
16.
2. Of the four choices gives after the statements,
choose the one that best describes the two B C
D E
Eb
statements.
A F
Statement-1
M
Energy is released when heavy nuclei undergo
fission or light nuclei undergo fusion. The above is a plot of binding energy per nucleon
Statement-2 Eb, against the nuclear mass M; A, B, C, D, E, F
correspond to different nuclei. Consider four
For heavy nuclei, binding energy per nucleon
increases with increasing Z while for light nuclei reactions :
it decreases with increasing Z. [AIEEE - 2008] (i) A + B ® C + e (ii) C ® A + B + e
(iii) D + E ® F + e (iv) F ® D + E + e
(1) Statement-1 is false, Statement-2 is true.
where e is the energy released ? In which reactions
(2) Statement-1 is true, Statement-2 is true;
is e positive ? [AIEEE - 2009]
Statement-2 is a correct explanation for
(1) (ii) and (iv) (2) (ii) and (iii)
Statement-1.
(3) (i) and (iv) (4) (i) and (iii)
(3) Statement-1 is true, Statement-2 is true; Directions : Questions number 17–18 are
Statement-2 is not a correct explanation for
based on the following paragraph.
Statement-1.
A nucleus of mass M + Dm is at rest and decays
(4) Statement-1 is true, Statement-2 is false.
M
into two daughter nuclei of equal mass each.
15. Assume that the nuclear binding energy per nucleon 2
(B/A) versus mass number (A) is as shown in the Speed of light is c.
figure. Use this plot to choose the correct choice(s) 17. The speed of daughter nuclei is:- [AIEEE-2010]

given below : [JEE-2008]


Dm Dm
(1) c (2) c
M + Dm M + Dm
B/A

8–
2Dm Dm
6– (3) c (4) c
M M
4–
18. The binding energy per nucleon for the parent
2– node06\B0AI-BO\Kota\JEE(MAIN)\Booster Course Sheet\Phy\Eng\Heat, Wave & Modern Physics\07-Nuclear Physics & Radioactivity.p65
nucleus is E1 an that for the daughter nuclei is E2.
0– Then :- [AIEEE-2010]

100 200 A
(1) E1 = 2E2 (2) E2 = 2E1
(3) E1 > E2 (4) E2 > E1
(1) Fusion of two nuclei with mass numbers lying
in the range of 1 < A < 50 will release energy 19. A radioactive nucleus (initial mass number A and
atomic number Z) emits 3 a-particles and 2
(2) Fusion of two nuclei with mass numbers lying
in the range of 51 < A < 100 will release positrons. The ratio of number of neutrons to that
energy of protons in the final nucleus will be[AIEEE-2010]
(3) Fission of a nucleus lying in the mass range of
A-Z-4 A - Z -8
100 < A < 200 will release energy when broken (1) (2)
into two equal fragments Z-2 Z-4

(4) Fission of a nucleus lying in the mass range of


200 < A < 260 will release energy when broken A-Z-4 A - Z - 12
(3) (4)
into two equal fragments Z -8 Z-4

46 E
Nuclear Physics & Radioactivity
ALLEN
20. When Uranium is bombarded with neutrons, it RADIOACTIVITY
undergoes fission. The fission reaction can be written
as : 23. A 280 days old radioactive substance shows an

U235 + 0n1 ® 56Ba141 + Kr92+ 3x + Q (energy) activity of 6000 dps, 140 days later it’s activity
92 36

where three particles named x are produced and becomes 3000dps. What was its initial activity.
energy Q is released . What is the name of the [JEE 2004 (Scr)]
particle x ? [JEE(Main)-2013 (Online)]
(1) 20000 dps
(1) neutrino (2) a-particle
(3) electron (4) neutron (2) 24000 dps
21. Match List I of the nuclear processes with List II (3) 12000 dps
containing parent nucleus and one of the end
products of each process and then select the correct (4) 6000 dps
answer using the codes given below the lists: 24. A radioactive sample S1 having an activity of 5µCi
[JEE Advance-2013]
has twice the number of nuclei as another sample
List I List II
S2 which has an activity of 10µCi. The half lives of
15
P. Alpha decay 1. 8 O ®15
7 N + ..... S1 and S2 can be : [JEE-2008]

238 234 (1) 20 years and 5 years, respectively


Q. b+ decay 2. 92 U ®90 Th + .....
(2) 20 years and 10 years, respectively
185
R. Fission 3. 83 Bi ®184
82 Pb + .....
(3) 10 years each
239
S. Proton emission 4. 94 Pu ®140
57 La + ..... (4) 5 years each
Codes : 25. To determine the half life of a radioactive element,
P Q R S
(1) 4 2 1 3 dN ( t )
a student plots a graph of ln versus t.
(2) 1 3 2 4 dt
(3) 2 1 4 3
(4) 4 3 2 1
dN ( t )
22. Match the nuclear processes given in column-I with Here is the rate of radioactive decay at
the appropriate option(s) in column-II.
dt
[JEE Advance-2015] time t. If the number of radioactive nuclei of this
Column-I Column-II element decreases by a factor of p after 4.16 years,
(A) Nuclear fusion (P) Absorption of thermal the value of p is
node06\B0AI-BO\Kota\JEE(MAIN)\Booster Course Sheet\Phy\Eng\Heat, Wave & Modern Physics\07-Nuclear Physics & Radioactivity.p65

[JEE 2010]
235
neutrons by 92 U

60
(B) Fission in a (Q) 27 Co
nucleus
nuclear reactor
(C) b-decay (R) Energy production in
stars via hydrogen
conversion to helium
(D) g-ray emission (S) Heavy water
(T) Neutrino emission
(1) (A)–P & S; (B)–R or R,T; (C)–Q & T; (D)–R
(2) (A)–R; (B)–P & S; (C)–Q & T; (D)–R or R,T
(1) 2 (2) 4
(3) (A)–R or R,T; (B)–P & S; (C)–Q & T; (D)–R
(3) 6 (4) 8
(4) (A)–R or R,T; (B)–Q & T; (C)–P & S; (D)–R

E 47
JEE ( Main) - Physics
ALLEN
Paragraph for Questions 26 and 27 28. Half-lives of two radioactive elements A and B are
The b-decay process, discovered around 1900, is 20 minutes and 40 minutes, respectively. Initially,
basically the decay of a neutron (n). In the the samples have equal number of nuclei. After 80

laboratory, a proton (p) and an electron (e ) are minutes , the ratio of decayed numbers of A and B
observed as the decay products of the neutron. nuclei will be :- [JEE-Main-2016]

Therefore, considering the decay of a neutron as a (1) 5 : 4 (2) 1 : 16


two-body decay process, it was predicted (3) 4 : 1 (4) 1 : 4
theoretically that the kinetic energy of the electron 29. A radioactive nucleus A with a half life T, decays
should be a constant. But experimentally, it was into a nucleus B. At t = 0, there is no nucleus B.
observed that the electron kinetic energy has a At sometime t, the ratio of the number of B to that
continuous spectrum. Considering a three-body of A is 0.3. Then, t is given by : [JEE-Main-2017]
r
decay process, i.e. n® p + e– + ve , around 1930,
T
Pauli explained the observed electron energy (1) t = T log (1.3) (2) t =
log(1.3)
r
spectrum. Assuming the anti-neutrino ( ve ) to be
massless and possessing negligible energy, and the T log2 log1.3
(3) t = (4) t = T
neutron to be at rest, momentum and energy 2 log1.3 log2
conservation principles are applied. From this
30. It is found that if a neutron suffers an elastic collinear
calculation, the maximum kinetic energy of the
collision with deuterium at rest, fractional loss of
electron is 0.8 × 106 eV. The kinetic energy carried
its energy is pd ; while for its similar collision with
by the proton is only the recoil energy.
carbon nucleus at rest, fractional loss of energy is
26. If the anti-neutrino had a mass of 3 eV/c 2
pc. The values of pd and pc are respectively :
(where c is the speed of light) instead of zero mass,
[JEE-Main-2018]
what should be the range of the kinetic energy, K,
(1) (.28, .89) (2) (0, 0)
of the electron? [JEE 2012]
(3) (0, 1) (4) (.89, .28)
(1) 0 £ K £ 0.8 ´ 10 6 eV

(2) 3.0 eV £ K £ 0.8 ´ 10 6 eV

(3) 3.0 eV £ K < 0.8 ´ 10 6 eV

(4) 0 £ K < 0.8 ´ 10 6 eV node06\B0AI-BO\Kota\JEE(MAIN)\Booster Course Sheet\Phy\Eng\Heat, Wave & Modern Physics\07-Nuclear Physics & Radioactivity.p65

27. What is the maximum energy of the anti-neutrino?


[JEE 2012]

(1) zero
6
(2) much less than 0.8 ×10 eV
(3) Nearly 0.8 × 106 eV
6
(4) Much larger than 0.8 × 10 eV

PREVIOUS YEARS QUESTIONS ANSWER KEY Exercise-II


Que. 1 2 3 4 5 6 7 8 9 10
Ans. 4 2 3 4 3 1 2 3 2 3
Que. 11 12 13 14 15 16 17 18 19 20
Ans. 2 2 1 4 2,4 3 3 4 3 4
Que. 21 22 23 24 25 26 27 28 29 30
Ans. 3 3 2 1 4 4 3 1 4 4

48 E
Semiconductor, POC & Practical Physics
ALLEN
SEMICONDUCTOR, POC & PRACTICAL PHYSICS EXERCISE-I

1. Which of the following statements is incorrect ? 6. The V-I characteristic for a p-n junction diode is
(1) Resistance of semiconductor decreases with plotted as shown in the figure. From the plot we
increase in temperature can conclude that

(2) In an electric field, displacement of holes is [Vb ® breakdown voltage, Vk ® knee voltage]
opposite to the displacement of electrons
(3) Resistance of a conductor decreases with the
increase in temperature
I (mA)
(4) n-type semiconductors are neutral
Vb
2. Pick out the wrong one for the statement. The
Vk V
reverse current in p-n junction diode
(1) is very small and constant
I (µA)
(2) remains constant even after the breakdown
voltage
(3) becomes infinity at breakdown (1) the forward bias resistance of diode is very high;
(4) reverse current is controlled by external almost infinity for small values of V and after a
resistance certain value it becomes very low
3. For the circuit shown in the figure:
(2) the reverse bias resistance of diode is very high
R=5kW
in the beginning upto breakdown voltage is not
70V
achieved
120V 10kW

(3) both forward and reverse bias resistances are


same for all voltages
(A) current through zener diode is 4 mA
(B) current through zener diode is 9 mA (4) both (1) and (2) are correct

(C) the output voltage is 50 V 7. The diode used in the circuit shown in the figure
(D) the output voltage is 40 V has a constant voltage drop of 0.5 V at all currents
Select the correct one and a maximum power rating of 100 milliwatts.
What should be the value of the resistor R connected
node06\2B0AH-AI\Kota\JEE(MAIN)\Crash Course Sheet\Phy\Eng\Heat, Wave & Modern Physics\08-Semiconductor, POC & Practical Physics.p65

(1) A & D (2) A & C (3) B & D (4) B & C


in series with the diode for obtaining maximum
4. Which of the following devices is heavily doped p-n
current -
junction comparitively ?
(1) Photo diode (2) Light emitting diode
R 0.5V
(3) Solar cell (4) Zener diode
5. Which of the following statements is correct for
proper working of zener diode ?
(1) Reverse bias voltage should be less than or 1.5V
equal to zener breakdown voltage
(2) Zener diode is to be reverse biased for zener (1) 1.5 W
action
(2) 5 W
(3) For given zener diode there can be different
(3) 6.67 W
zener breakdown voltages
(4) 200 W
(4) All of above

E 49
JEE ( Main) - Physics
ALLEN
8. In the following circuits PN-junction diodes D1, D2 12. A common emitter amplifier is designed with NPN
and D3 are ideal for the following potential of A transistor (a = 0.99). The input impedance is 1KW
and B, the correct increasing order of resistance and load is 10 KW. The voltage gain will be
between A and B will be -
(1) 9.9 (2) 99 (3) 990 (4) 9900
13. In a common amitter transistor amplifier, when
D1 R
signal voltage changes by 0.01 volt, the base
D2 R current changes by 10 mA and collector current
changes by 1 mA. If the collector resistance
R D3 R R
4 RC = 4 kW and load resistance RL = 8 kW, then the
4
A B current gain is
(1) 1 (2) 10 (3) 100 (4) 1000
(i) –10V, –5V, (ii) –5V, –10V
14. Figure shows the transfer characteristics of a base
(iii) –4V, –12V biased CE transistor. Which of the following
statements is FALSE ?
(1) (i) < (ii) < (iii) (2) (iii) < (ii) < (i)
(3) (ii) = (iii) < (i) (4) (i) = (iii) < (ii) V0

9. A sinusoidal voltage of peak value 200 volt is


connected to a diode and resistor R in the circuit
shown so that half wave rectification occurs. If the
forward resistance of the diode is negligible
compared to R then rms voltage (in volt) across R is
approximately - 0 0.6V 2V Vin

(1) At Vi = 1V , it can be used as an amplifier.

E0 = 200 Volt R (2) At Vi = 0.5V, it can be used as a switch turned


off.
(3) At Vi = 2.5V, it can be used as a switch turned
on.
200
(1) 200 (2) 100 (3) (4) 280 (4) At Vi = 0.4V, transistor is in active state.
2
15. In given transistor circuit at saturation mode
10. The mobility of electrons and holes in a sample of VBE = 0.8V, VCE = 0.2V and b = 100 then find the
node06\2B0AH-AI\Kota\JEE(MAIN)\Crash Course Sheet\Phy\Eng\Heat, Wave & Modern Physics\08-Semiconductor, POC & Practical Physics.p65
instrinsic germanium at room temperature are 0.36 maximum value of RC for which transister remains
and 0.14 m2/V-s. If electron and holes densities each in saturation mode :
are equal to 2.5 ×1019/m3 then conductivity of germanium
will be (in ohm meter) : 10V
(1) 2 (2) 4 RC
(3) 6 (4) 8 C
200 kW
11. In an N-P-N transistor, the collector current is
B
10 mA, if 80% emitted electrons reach the collector
5V E
then :-
(1) the emitter current will be 7.5 mA
(2) the emitter current will be 12.5 mA
(3) the base current will be 3.5 mA (1) 2.31 kW (2) 4.67 kW
(4) the base current will be 1.5 mA (3) 5.32 kW (4) 6.24 kW

50 E
Semiconductor, POC & Practical Physics
ALLEN
16. Three amplifiers X, Y and Z are connected in series. 18. A combination of logic gates has the truth table
If the voltage gains of X, Y and Z are below.
10, 20 and 30 respectively and the input signal is P Q Z
1 mV peak value. Then what is the output signal
0 0 0
voltage peak value if dc supply voltage is 5V :-
0 1 1
(1) 4 V (2) 5 V
1 0 1
(3) 6 V (4) 7 V
1 1 1
17. Input waveforms A and B as shown in fig I are
Which of the following combinations has this truth
applied to the combination of gates as shown in
table ?
Fig.II. Which of the waveforms shown in Fig. (i) to
(iv) correctly represents the output waveform?
P
(1) z

A Q

P
B z
(2)
Fig. I
Q

A P
Y z
(3)
B Q

Fig-II
P
z
(4)
(1) Q
node06\2B0AH-AI\Kota\JEE(MAIN)\Crash Course Sheet\Phy\Eng\Heat, Wave & Modern Physics\08-Semiconductor, POC & Practical Physics.p65

Fig. (i)
19. Output Y of the given logic gate network is :

(2) A
Y
Fig. (ii)

(3)
(1) A.B + A.B
Fig. (iii)

(2) A.B + A.B

(4) ( )
(3) A + B . A . B

Fig. (iv)
(4) None
E 51
JEE ( Main) - Physics
ALLEN
20. Truth table for the given circuit is :- 24. In modulation process :-
(1) Characteristic of carrier signal is varied in
A
accordance with the instantaneous value of the
Y message signal
B
(2) Characteristic of message signal is varied in
accordance with the instantaneous value of the
A B Y carrier signal
(1) (2) A B Y
0 0 1 0 0 0 (3) Characteristic of carrier signal is varied in
0 1 0 0 1 0 accordance with the average value of the
1 0 1 1 0 0 message signal
1 1 0 1 1 0
(4) None of these
25. A carrier signal 60sin(2p ×106 t) is used to modulate
(3) A B Y (4) A B Y aud io sig nal 15sin(300pt) . The depth of
0 0 0 0 0 1 modulation is :
0 1 1 0 1 1
(1) 50 % (2) 40%
1 0 0 1 0 1
1 1 1 1 1 1 (3) 25% (4) 15%
26. Total power of amplitude modulated wave is given
21. Electromagnetic wave with frequencies greater than by :-
the critical frequency of ionosphere cannot be used
for communication using sky wave propagation, Vm2 é m 2 ù
because :- (1) ê1 + ú
2R ë 2û
(1) The refractive index of the ionosphere becomes
very high for f > fC Vc2 é m 2 ù
(2) ê1 + ú
(2) The refractive index of the ionosphere becomes 2R ë 2û
very low for f > fC

(3) The refractive index of the ionosphere becomes Vc2 m 2


(3)
very high for f < fC 2R 2
(4) None of these

node06\2B0AH-AI\Kota\JEE(MAIN)\Crash Course Sheet\Phy\Eng\Heat, Wave & Modern Physics\08-Semiconductor, POC & Practical Physics.p65
22. A transmitting antenna of height 20m and the
Vm VC é m 2 ù
(4) ê1 + ú
receiving antenna of height h are separated by a 2R ë 2û
distance of 40 km for satisfactory communication in
27. The diameter of a cylinder is measured using a
line of sight mode, then the value of h is :-
Vernier callipers with no zero error. It is found that
(1) 40m (2) 45m the zero of the Vernier scale lies between 5.10 cm
and 5.15 cm of the main scale. The Vernier scale
(3) 30m (4) 25m
has 50 divisions equivalent to 2.45 cm. The 24th
23. A ground receiver in line of sight communication can division of the Vernier scale excatly coincides with
not receive direct waves due to :- one of the main scale divisions. The diameter of
the cylinder is :-
(1) its low frequency
(1) 5.112 cm
(2) curvature of earth
(2) 5.124 cm
(3) its high frequency
(3) 5.136 cm
(4) smaller antenna
(4) 5.148 cm
52 E
Semiconductor, POC & Practical Physics
ALLEN
28. Consider a Vernier callipers in which each 1 cm on 29. When the gap is closed without placing any object
the main scale is divided into 8 equal divisions and in the screw gauge whose least count is 0.005 mm,
a screw gauge with 100 divisions on its circular scale. the 5th division on its circular scale coincides with
In the Vernier callipers, 5 divisions of the Vernier the reference line on main scale, and when a small
scale coincide with 4 divisions on the main scale sphere is placed reading on main scale advances
and in the screw gauge, one complete rotation of by 4 divisions, whereas circular scale reading
the circular scale moves it by two divisions on the advances by five times to the corresponding reading
linear scale. Then : when no object was placed. There are 200 divisions
on the circular scale. The radius of the sphere is
(1) If the pitch of the screw gauge is twice the least
count of the Vernier callipers, the least count (1) 4.10 mm (2) 4.05 mm
of the screw gauge is 0.01 mm.
(3) 2.10 mm (4) 2.05 mm
(2) If the pitch of the screw gauge is twice the least
30. A vern ier callipers h aving 1 main scale
count of the Vernier callipers, the least count
division = 0.1 cm is designed to have a least count
of the screw gauge is 0.005 mm.
of 0.02 cm. If n be the number of divisions on vernier
(3) If the least count of the linear scale of the screw scale and m be the length of vernier scale, then :-
gauge is twice the least count of the Vernier
(1) n = 10, m = 0.5 cm
callipers, the least count of the screw gauge is
0.01 mm. (2) n = 9, m = 0.4 cm

(4) If the least count of the linear scale of the screw (3) n = 10, m = 0.8 cm
gauge is twice the least count of the Vernier (4) n = 10, m = 0.2 cm
callipers, the least count of the screw gauge is
0.005 mm.
node06\2B0AH-AI\Kota\JEE(MAIN)\Crash Course Sheet\Phy\Eng\Heat, Wave & Modern Physics\08-Semiconductor, POC & Practical Physics.p65

ANSWER KEY Exercise-I


Que. 1 2 3 4 5 6 7 8 9 10
Ans. 3 2 4 4 2 4 2 3 2 1
Que. 11 12 13 14 15 16 17 18 19 20
Ans. 2 3 3 4 2 3 3 3 4 3
Que. 21 22 23 24 25 26 27 28 29 30
Ans. 1 2 2 1 3 2 2 2,3 4 3
E 53
JEE ( Main) - Physics
ALLEN
PREVIOUS YEARS' QUESTIONS EXERCISE-II
1. In a common-base mode of a transistor, the 5. In the circuit below, A and B represent two inputs
collector current is 5.488 mA for an emitter current and C represents the output. The circuit represents
of 5.60 mA. The value of the base current [AIEEE - 2008]
amplification factor (b) will be- [AIEEE - 2006]
A
(1) 49 (2) 50 C
B
(3) 51 (4) 48

2. If the ratio of the concentration of electrons to that

7 (1) NOR gate (2) AND gate


of holes in a semiconductor is and the ratio of
5
(3) NAND gate (4) OR gate
7
currents is , then what is the ratio of their drift 6. Two full turns of the circular scale of screw gauge
4 cover a distance of 1 mm on scale. The total number
velocities ? [AIEEE - 2006] of divisions on circular scale is 50. Further, it is found
that screw gauge has a zero error of -0.03 mm.
5 4 5 4 While measuring the diameter of a thin wire a student
(1) (2) (3) (4)
8 5 4 7 notes the main scale reading of 3 mm and the
number of circular scale division in line, with the
3. The circuit has two oppsitely connected ideal diodes main scale is 35. The diameter of the wire is :-
in parallel. What is the current flowing in the circuit
[AIEEE - 2008]
[AIEEE - 2006]
(1) 3.32 mm (2) 3.73 mm
(3) 3.67 mm (4) 3.38 mm
4W
7. A p-n junction (D) shown in the figure can act as a
D1 D2 rectifier. An alternating current source (V) is
connected in the circuit.
12V 3W 2W

(1) 1.71 A (2) 2.00 A v ~ R

(3) 2.31 A (4) 1.33 A


The current (I) in the resistor (R) can be shown node06\2B0AH-AI\Kota\JEE(MAIN)\Crash Course Sheet\Phy\Eng\Heat, Wave & Modern Physics\08-Semiconductor, POC & Practical Physics.p65
4. A working transitor with its three legs marked P,
by :- [AIEEE - 2009]
Q and R is tested using a multimeter. No
conduction is found betwen P and Q. By connecting I

the common (negative) terminal of the multimeter (1)


t
to R and the other (positive) terminal to P or Q,
some resistance is seen on the multimeter. Which I

of the following is true for the transistor? (2)


t
[AIEEE - 2008]
I
(1) It is an npn transistor with R as base
(3)
t
(2) It is a pnp transistor with R as collector

(3) It is a pnp transistor with R as emitter I


(4)
(4) It is an npn transistor with R as collector t
54 E
Semiconductor, POC & Practical Physics
ALLEN
8. The logic circuit shown below has the input waveform 11. This question h as Statement -1 and
'A' and 'B' as shown. Pick out the correct output Statement-2. Of the four choices given after the
waveform :- statements, choose the one that best describes the
two statements.
A Statement-1:
Y
Sky wave signals are used for long distance radio
B
communication. These signals are in general, less
stable than ground wave signals.
Input A
Statement-2 :
Input B
The state of ionosphere varies from hour to hour,
day to day and season to season. [AIEEE - 2011]
Output is :- [AIEEE - 2009]
(1) Statement-1 is true, Statement-2 is true and
Statement-2 is not the correct explanation
(1) of Statement-1.
(2) Statement-1 is false, Statement-2 is true
(3) Statement-1 is true, Statement-2 is false
(2)
(4) Statement-1 is true, Statement-2 is true and
Statement-2 is the correct explanation of
statement-1.
(3)
12. The ouput of an OR gate is connected to both the
inputs of a NAND gate. The combination will serve
as a : [AIEEE - 2011]
(4)
(1) OR gate (2) NOT gate
9. In an experiment the angles are required to be (3) NOR gate (4) AND gate
measured using an instrument, 29 divisions of the 13. Which of the following four alternatives is not correct
main scale exactly coincide with the 30 divisions of We need modulation :-
the vernier scale. If the smallest division of the main [AIEEE - 2011]
scale is half-a-degree (=0.5°), then the least count (1) To increase the selectivity
of the instrument is :- [AIEEE-2009]
(2) To reduce the time lag between transmission and
(1) One degree reception of the information signal
node06\2B0AH-AI\Kota\JEE(MAIN)\Crash Course Sheet\Phy\Eng\Heat, Wave & Modern Physics\08-Semiconductor, POC & Practical Physics.p65

(3) to reduce the size of antenna


(2) Half degree
(4) To reduce the fractional band width, that is the
(3) One minute ratio of the signal band width to the centre
(4) Half minute frequency
14. A screw gauge gives the following reading when used
10. The combination of gates shown below yields:-
to measure the diameter of a wire.
[AIEEE - 2010] Main scale reading : 0 mm.
Circular scale reading : 52 divisions
Given that 1 mm on main scale corresponds to 100
divisions of the circular scale.
The diameter of wire from the above date is :-
[AIEEE - 2011]
(1) NAND gate (2) OR gate
(1) 0.026 cm (2) 0.005 cm
(3) NOT gate (4) XOR gate (3) 0.52 cm (4) 0.052 cm
E 55
JEE ( Main) - Physics
ALLEN
15. Truth table for system of four NAND gates as shown 18. The I-V characteristic of an LED is :
in figure is :- [AIEEE - 2012] [JEE (Main) 2013]

Yellow
Green
Blue
Red
Y I
I (R) (Y) (G) (B) (B)
B (G)
(Y)
(R)
(1) (2)

A B Y A B Y O V
O V
0 0 1 0 0 0
0 1 0 0 1 1
(1) (2)
1 0 0 1 0 1
1 1 1 1 1 0 V O
I
(R)

(3) (4) (Y)

A B Y A B Y
(G)
(B)
I
0 0 0 0 0 1
O V
0 1 0 0 1 1
(3) (4) 19. Which of the following circuits correctly represents
1 0 1 1 0 0
1 1 1 1 1 0 the following truth table ? [JEE (Main) 2013(Online)]

A B C
16. A spectrometer gives the following reading when
0 0 0
use d to mea sure the ang le o f a prism.
0 1 0
Main scale reading : 58.5 degree [AIEEE - 2012]
1 0 1
Vernier scale reading : 09 divisions Given that 1 1 1 0
division on main scale corresponds to 0.5 degree.
Total divisions on the vernier scale is 30 and match
A
with 29 divisions of the main scale. The angle of the
(1) B C
prism from the above data :

(1) 59 degree

(2) 58.59 degree node06\2B0AH-AI\Kota\JEE(MAIN)\Crash Course Sheet\Phy\Eng\Heat, Wave & Modern Physics\08-Semiconductor, POC & Practical Physics.p65
(2) A C
(3) 58.77 degree B

(4) 58.65 degree A


17. N divisions on the main scale of a vernier callipers (3) B C
coincide with (N + 1) divisions of the vernier, scale. If
each division of main scale is 'a' units, then the least A
(4) C
count of the instrument is :- B
[AIEEE - 2012 (Online)] 20. Which of the following modulated signal has the best
noise-tolerance ? [JEE (Main) 2013(Online)]
a N
(1) (2) ´a (1) short-wave
N N +1
(2) medium-wave
a (3) long-wave
(3) a (4)
N +1 (4) amplitude-modulated

56 E
Semiconductor, POC & Practical Physics
ALLEN
21. If a carrier wave c(t) = A sin wct, were to be 25. Choose the correct statement : [JEE (Main) 2016]

amplitude modulated by a modulating signal (1) In frequency modulation the amplitude of the
m(t) = A sinwmt, the equation representing the high frequency carrier wave is made to vary in
modulated signal [Cm(t)], and its modulation index, proportion to the frequency of the audio signal.
would be respectively : [JEE (Main) 2013(Online)] (2) In amplitude modulation the amplitude of the
(1) Cm(t) = A (1 + sinwmt) sinwct and 1 high frequency carrier wave is made to vary in
proportion to the amplitude of the audio signal.
(2) Cm(t) = A (1 + sinwmt) sinwct and 2
(3) In amplitude modulation the frequency of the
(3) Cm(t) = A (1 + sinwct) sinwmt and 2 high frequency carrier wave is made to vary in
proportion to the amplitude of the audio signal.
(4) Cm(t) = A (1 + sinwct) sinwmt and 1
(4) In frequency modulation the amplitude of the
22. The forward biased diode connection is : high frequency carrier wave is made to vary in
[JEE (Main) 2014] proportion to the amplitude of the audio signal.

26. For a common emitter configuration, if a and b have


2V 4V
(1) their usual meanings, the incorrect relationship
between a and b is [JEE (Main) 2016]
–2V
(2)
b2 1 1
(1) a = (2) = +1
+2V 1 + b2 a b
(3)

–3V b b
(4) (3) a = (4) a =
1- b 1+ b
23. A student measured the length of a rod and wrote
27. If a, b, c, d are inputs to a gate and x is its output,
it as 3.50 cm. Which instrument did he use to
then as per the following time graph, the gate is
measure it? [JEE (Main) - 2014]
(d) [JEE (Main) 2016]
(1) A screw gauge having 100 divisions in the
circular scale and pitch as 1 mm. (c)

(2) A screw gauge having 50 divisions in the circular (b)


scale and pitch as 1 mm.
node06\2B0AH-AI\Kota\JEE(MAIN)\Crash Course Sheet\Phy\Eng\Heat, Wave & Modern Physics\08-Semiconductor, POC & Practical Physics.p65

(a)
(3) A meter scale
(x)
(4) A vernier calliper where the 10 divisions in
vernier scale matches with 9 division in main (1) NAND (2) NOT
scale and main scale has 10 divisions in 1 cm. (3) AND (4) OR
24. A single of 5 kHz frequency is amplitude modulated 28. The temperature dependence of resistances of Cu
on a carrier wave of frequency 2 MHz. The and undoped Si in the temperature range
frequencies of the resultant signal is/are - 300-400K, is best described by :- [JEE (Main) 2016]

[JEE (Main) 2015] (1) Linear decrease for Cu, linear decrease for Si.

(1) 2005 kHz, 2000 kHz and 1995 kHz (2) Linear increase for Cu, linear increase for Si.

(2) 2000 kHz and 1995 kHz (3) Linear increase for Cu, exponential increase
for Si
(3) 2 MHz only
(4) Linear increase for Cu, exponential decrease for
(4) 2005 kHz and 1995 kHz Si
E 57
JEE ( Main) - Physics
ALLEN
29. Identify the semiconductor devices whose 30. A screw gauge with a pitch of 0.5 mm and a circular
characteristics are given below, in the order (a), (b), scale with 50 divisions is used to measure the thickness
(c), (d) :- [JEE (Main) 2016] of a thin sheet of Aluminium. Before starting the
measurement , it is found that when the two jaws of
I I the screw gauge are brought in contact, the 45th
division coincides with the main scale line and that
(a) V (b) V the zero of the main scale is barely visible. What is
the thickness of the sheet if the main scale reading
is 0.5 mm and the 25th division coincides with the
main scale line ? [JEE(Main)-2016]

Resistance (1) 0.50 mm (2) 0.75 mm


I dark
(3) 0.80 mm (4) 0.70 mm

(c) V (d)
Intensity
Illuminated of light

(1) Zener diode, Solar cell, Simple diode, Light


dependent resistance
(2) Simple diode, Zener diode, Solar cell, Light
dependent resistance
(3) Zener diode, Simple diode, Light dependent
resistance, Solar cell
(4) Solar cell, Light dependent resistance, Zener
diode, Simple diode

node06\2B0AH-AI\Kota\JEE(MAIN)\Crash Course Sheet\Phy\Eng\Heat, Wave & Modern Physics\08-Semiconductor, POC & Practical Physics.p65

PREVIOUS YEARS QUESTIONS ANSWER KEY Exercise-II


Que. 1 2 3 4 5 6 7 8 9 10
Ans. 1 3 2 1 4 4 1 3 3 2
Que. 11 12 13 14 15 16 17 18 19 20
Ans. 1 3 2 4 2 4 4 1 2 1
Que. 21 22 23 24 25 26 27 28 29 30
Ans. 1 3 4 1 2 1,3 4 4 2 3

58 E

You might also like